Vous êtes sur la page 1sur 55

A-PDF N-up Page Demo. Purchase from www.A-PDF.

com to remove the watermark

2
Section A

NANYANG JUNIOR COLLEGE


JC 2 PRELIMINARY EXAMINATION
Higher 2

CHEMISTRY

For each question there are four possible answers, A, B, C and D. Choose the one
you consider to be correct.

9647/01

29 September 2015

Paper 1 Multiple Choice

1 hour
Additional Materials:

A 20 cm3 sample of methanol is ignited with excess oxygen. The volume of


the residual gas obtained was 80 cm 3. When the residual gas was shaken
with aqueous sodium hydroxide, the volume decrease by 20 cm 3. What is the
initial volume of oxygen present? (All volumes are measured at room
temperature and pressure.)

Multiple Choice Answer Sheet


Data Booklet

60 cm3

80 cm3

90 cm3

100 cm3

READ THESE INSTRUCTIONS FIRST


Write in soft pencil.
Do not use staples, paper clips, highlighters, glue or correction fluid.
Write your name, class and tutors name on the Answer Sheet in the spaces provided
unless this has been done for you.

Consider the following half-equations.


Cr2O72 + 14H+ + 6e 2Cr3+ + 7H2O

There are forty questions on this paper. Answer all questions. For each question there are
four possible answers A, B, C and D.
Choose the one you consider correct and record you choice in soft pencil on the separate
Answer Sheet.

Fe2+ Fe3+ + e
C2O42 2CO2 + 2e

Read the instructions on the Answer Sheet very carefully.


Each correct answer will score one mark. A mark will not be deducted for a wrong answer.
Any rough working should be done in this booklet.

What volume of 0.02 mol dm3 Na2Cr2O7 is required to oxidise 20 cm3 of an


acidified solution of 0.03 mol dm3 FeC2O4?
A

5 cm3

10 cm3

15 cm3

This document consists of 19 printed pages.


[Turn over

H2 Chemistry 9647/01 NYJC J2/2015 Prelim

30 cm3

3
3

The use of the Data Booklet is relevant to this question.

A sample of the element silicon (Si) was vaporised, ionised and passed
through an electric field. It was observed that a beam of 30Si2+ particles gave
an angle of deflection of +2.

30

Si2+

Beryllium dichloride, BeCl2, reacts with methylamine, CH3NH2 to form a


compound. Which one of the statements is incorrect?
A

The compound is formed from 1 mole of BeCl2 and 2 moles of CH3NH2.

The Be-N bond formed is chemically similar to a covalent bond.

The compound is capable of forming only two hydrogen bonds per


molecule.

The beryllium atom in beryllium dichloride is electron deficient.

+2

+
Assuming an identical set of experimental conditions, by what angle would the
following sample of arsenic be deflected?
Isotope
73

As

No. of electrons

Flask A contains 1 dm3 of neon at 2 kPa and flask B contains 2 dm3 of argon
at 1 kPa. If the flasks are connected at constant temperature, what is the final
pressure?
A

36

1.33 kPa

1.5 kPa

1.67 kPa

2 kPa

Angle of deflection for 73As


A

+3.2

3.2

+1.2

1.2

The use of the Data Booklet is relevant to this question.


Aluminium oxide is a stable ionic compound.
Al (s) Al3+ (g) + 3e
O2 (g) + 2e O2- (g)

H = +5467 kJ mol-1
H = +897 kJ mol-1

Which of the following statements is incorrect?

Which of the following pairs have the same shape?


A

PCl3

BF3

NO2+

HNO2

SnCl2

HCN

SO32-

NH3

H2 Chemistry 9647/01 NYJC J2/2015 Prelim

[Turn over

The enthalpy change of atomisation of aluminium is +330 kJ mol-1.

The enthalpy change


13625 kJ mol-1.

The lattice energy of aluminium oxide is more exothermic than the


enthalpy change of formation of aluminium oxide.

The sum of the first and second electron affinities of oxygen is


+649 kJ mol-1.

of

formation

H2 Chemistry 9647/01 NYJC J2/2015 Prelim

of

aluminium

oxide

is

5
8

Given the following reaction and its rate law,

10

2A + B 2C
Rate = k[A][B]

One type of rechargeable battery makes use of the nickel-cadmium cell, in


which nickel and cadmium electrodes, coated with the respective hydroxides,
are immersed in potassium hydroxide solution. During discharge, the
cadmium electrode releases electrons to the external circuit.

which of the following graphs correctly represents the rate law?

The relevant electrodes are:

Cd(OH)2(s) + 2e Cd(s) + 2OH(aq)


NiO(OH)(s) + H2O(l) + e Ni(OH)2(s) + OH(aq)

B
rate

[A]
0.01

[A] = 0.01 mol dm-3


[B] = 0.01 mol dm-3

Which of the following is incorrect about this cell during discharge?

0.005
0.0025
x

[A][B]

rate

2x

Time

[C]

The concentration of the potassium hydroxide solution remains


constant.

The mass of the cadmium electrode decreases as cadmium is used up.

Cadmium is the negative electrode.

The electron flows from cadmium electrode to nickel electrode in the


external circuit.

0.02
0.015
[A] = 1.00 mol dm-3
[B] = 0.01 mol dm-3

0.01

[B]

x 2x

11

0.1091

0.1575

0.5040

The ratio of chlorine : hydrogen : sodium hydroxide formed is 1:2:1.

Sodium ions migrate through the diaphragm from cathode to anode.

When the diaphragm is removed, sodium chlorate(V) is formed at room


temperature.

Chlorine is produced at the anode while the pH of the electrolyte


around the cathode increases.

Time

The decomposition of a sample of H2O2 solution is a first order reaction with a


rate constant of 7.70 x 10-4 s-1. What is the initial concentration in mol dm-3 of
the H2O2 solution if its concentration has dropped to 0.0315 mol dm -3 in
75 min?
A

Which of the following statements about the electrolysis of brine using a


diaphragm cell is correct?

1.008
12

At 450 C, 50 % of M decomposes according to the equation:


M(g) 2N(g) + Q(s)
If the equilibrium pressure is 2 atm, what is the equilibrium constant, Kp for the
reaction at this temperature?
A

H2 Chemistry 9647/01 NYJC J2/2015 Prelim

[Turn over

0.148

H2 Chemistry 9647/01 NYJC J2/2015 Prelim

2.67

7
13

The titration curve below shows the reaction between a 20.0 cm3 of
0.20 mol dm3 of an acid, against 0.10 mol dm3 of base.

15

The graph below shows the variation in the enthalpy change of vaporisation,
Hvap for 8 consecutive elements in the Periodic Table, all with atomic
number 20.

pH

Hvap / kJ mol1
B

H
G
D
40
Volume of 0.10 mol dm

C
3

base added / cm

F
Atomic number

What can be deduced from the above graph?


Which of the following sets of reagents could give this curve?

14

I
II
III
IV

H2SO4 and NaOH


CH3COOH and KOH
4-hydroxybenzoic acid and NaOH
benzoic acid and NaOH

I only

I & III

II & IV

II, III & IV

16

A sparingly soluble aluminium salt dissociates in aqueous solution according


to the equation given.
Al2X3 (s) 2Al

3+

E is soluble in warm benzene whereas F is not.

The chlorides become less acidic from A to C.

The pH of the solution containing a mixture of oxide of G and oxide of


D is greater than 7.

The oxide of A reacts with excess aqueous sodium hydroxide to form a


soluble complex.

P, Q and R are Group II elements. They form compounds with the following
properties:

P(NO3)2 has a higher thermal decomposition temperature than


R(NO3)2.

RCO3 has a more endothermic


decomposition than QCO3.

enthalpy

Which of the following statements is correct?

2-

(aq) + 3X (aq)

If the solubility product, Ksp, of Al2X3 is S, what is the value of [Al3+ (aq)] at
equilibrium?

PO has a more exothermic lattice energy than RO.

QCl2 has more covalent character than PCl2.

R is more reducing than P.

Q is Mg and R is Ba.

1
2

2 2
S
5

1
5

8 5
S

27

H2 Chemistry 9647/01 NYJC J2/2015 Prelim

H2 Chemistry 9647/01 NYJC J2/2015 Prelim

[Turn over

change

of

9
17

10

When crystalline potassium chromate(VI) was dissolved in water, a yellow


solution T was formed. Addition of dilute sulfuric acid to T gave an orange
solution U. When hydrogen sulfide was bubbled through solution U, there was
a color change in the solution and yellow sulfur was produced.

20
OH
OH

Which process did not occur in the above experiment?


A

ligand exchange

precipitation

redox reaction

acid-base

HO

Q
Q is reacted with acidified hot concentrated manganate(VII) ions.

18

How many stereoisomers and oxygen atoms will be present in the organic
product(s) formed?

The compound 1,2-diaminoethane, H2NCH2CH2NH2, is a bidentate ligand; in


formulae, it is usually abbreviated to en. When 1,2-diaminoethane is added to
[Co(NH3)6]2+ in aqueous solution, [Co(en)3]2+ is formed. What is the best
explanation for this?
A

There are much stronger bonds between the ligands and the cobalt(II)
ion in [Co(en)3]2+ than in [Co(NH3)6]2+.
When [Co(en)3]2+ is formed from [Co(NH3)6]2+ the reaction is
exothermic.

When [Co(en)3]2+ is formed from [Co(NH3)6]2+ the entropy change is


positive.

No. of oxygen atoms

22

22

23

23

When [Co(en)3]2+ is formed from [Co(NH3)6]2+ the reaction has a low


activation energy.

21
19

No. of stereoisomers

An unknown compound has the following features:

Which bond is not present in HCCCH=C=CHCH2CH3?

It is non-cyclic.
It has a relative molecular mass of x.
It produces a silver mirror with Tollens Reagent.
It reacts with H2 in the presence of Ni to form a product with the
molecular mass of (x + 4).

What is the smallest possible value of the molecular mass, x?


A

54

56

58

60

H2 Chemistry 9647/01 NYJC J2/2015 Prelim

Covalent bonds are formed by orbital overlap. The shape of unsaturated


hydrocarbon molecules can be explained in terms of hybridisation of orbitals.

[Turn over

a bond formed by 2p 2p overlap

a bond formed by 1s 2sp2 overlap

a bond formed by 2sp 2s overlap

a bond formed by 2sp2 2sp overlap

H2 Chemistry 9647/01 NYJC J2/2015 Prelim

11
22

12

In the free radical substitution of 2,3-dimethylpentane with chlorine, a mixture


of mono-chlorinated compounds were obtained.

24

What type of reaction is not involved in the formation of the organic product
below?

How many different structural isomers can be formed?

N Na

NH

O
+

NCH 2 CH 3

CH3CH2Br

NaOH
O

KOH
O

23

A sequence of reactions is shown below.


Br

step 1

CH 3 CH 2 NH 2

step 2

O
OH

step 3

Br2 in CCl4

redox

neutralisation

hydrolysis

nucleophilic substitution

Br
Br

25

Which of the following shows the correct list of reagents and conditions for the
sequence?

One mole of compound T is warmed with excess ethanolic silver nitrate.


How many moles of silver chloride will be precipitated?

CH 2 Cl
step 1

step 2

step 3

AlBr3

NaBH4

Ethanolic NaOH

AlBr3

Na in ethanol

Al2O3

LiAlH4

Acidified K2Cr2O7

conc. H2SO4

LiAlH4

H2 with Ni

Al2O3

Cl
CH=CHCl

COCl

H2 Chemistry 9647/01 NYJC J2/2015 Prelim

H2 Chemistry 9647/01 NYJC J2/2015 Prelim

[Turn over

13
26

14

Which reaction could be used as a possible synthesis of this compound?

28

A compound R, C13H13O4Br undergoes the following reactions:

1 mole of R reacts with Tollens Reagent but not Fehlings solution to


produce 4 moles of silver.
R reacts with hot ethanolic sodium hydroxide to form 3 isomeric
products.
R reacts with hot acidified potassium dichromate to form two organic
compounds.

What is the structure of R?


CH3COOH

NH2

NH2

(CH3)2CHBr

CHO

A
(CH3)2CHBr

CH3COOH

COOCH 3

CHO
CH3COBr

NH2

(CH3)2CHBr

CH2 CHBrCH 3

COCHO

CHO

CH3

CH3COBr

NH2

OCOCH 3

27

II

CH3CH2NH2

CH3CON(CH3)2

CHO

CHBrCH 2 CH 3

In which pair of molecules is the pKb of species I bigger than that of species II ?
I

COOCH 3

CHO

CHBrCH 2 CH3

(CH3)2CHBr

C
D

CHO

COOH
CH 2 CHBrCH 3

29
CH 2 CN
COOH

B
O

O
HO

CH3COO

CH2ClCOO

CH=CH 2

D
NH2

H3 C

H2 Chemistry 9647/01 NYJC J2/2015 Prelim

NH2

How many moles of HCl(g) are required to react with one mole of the above
molecule?

[Turn over

H2 Chemistry 9647/01 NYJC J2/2015 Prelim

15
30

16

Denaturation of proteins occurs due to the disruption of the tertiary and


quaternary structures where various R group interactions are destroyed by
external factors such as pH changes and heavy metal ions.
Which of the following incorrectly shows the R group interactions by such
external factors?

Section B
For each of the questions in this section one or more of the three numbered
statements 1 to 3 may be correct.
Decide whether each of the statements is or is not correct (you may find it helpful to
put a tick against the statements which you consider to be correct).
The responses A to D should be selected on the basis of

R group interactions

External factor

hydrogen bonds

heating

van der Waals

extreme pH

disulfide bridges

heavy metal ions

No other combination of statements is used as a correct response.

ionic interactions

heavy metal ions

31

A
1, 2 and 3
are
correct

B
1 and 2
only are
correct

C
2 and 3
only are
correct

D
1 only
is
correct

Three organic molecules each have

three elements
the composition, by mass, C, 54.5%; H, 9.1%

What could these molecules be?

32

33

H2 Chemistry 9647/01 NYJC J2/2015 Prelim

[Turn over

CH3CH2CH2CO2H

OHCCH2CH2CH2OH

CH3CH=CHCH2SH

When a sample of gas is compressed at constant temperature from 20 atm to


50 atm, its volume changes from 96 cm3 to 46 cm3. Which of the following
statements is a possible explanation of this behaviour?
1

The gas particles are in a state of continual, random motion.

The gas particles have a finite volume.

There are forces of attraction between gas particles.

Which of the following relationships are correct?


1

2 Hformation(HI) = BE (HH) + BE (II)

2 Hatomisation(I2) = Hsublimation(I2) + BE (II)

Hformation(H2O) = Hcombustion(H2)

H2 Chemistry 9647/01 NYJC J2/2015 Prelim

17

18

The responses A to D should be selected on the basis of

A
B
C
1, 2 and 3
are
correct

1 and 2
only are
correct

2 and 3
only are
correct

The responses A to D should be selected on the basis of

A
B
C

D
1 only
is
correct

1, 2 and 3
are
correct

1 and 2
only are
correct

2 and 3
only are
correct

D
1 only
is
correct

No other combination of statements is used as a correct response.

No other combination of statements is used as a correct response.

34

35

In a chemical reaction, J reacts with K to form L. The initial rates of the


reaction are measured in the following experiment.
Experiment

[J]/ mol dm3

[K]/ mol dm3

Initial rate/ mol dm3

0.150

0.250

2.80 x 105

0.150

0.500

5.60 x 105

3
4

0.075
0.075

0.500
0.250

2.80 x 105
1.40 x 105

50 cm3 of 1 mol dm3 HCl and 50 cm3 of 1 mol dm3 KOH

The energy profile diagram for the reaction is as shown:


Energy

Which of these pairs of substances, when mixed, would lead to the same
temperature rise as the reaction below?

36

50 cm3 of 0.5 mol dm3 H2SO4 and 50 cm3 of 1 mol dm3 KOH

25 cm3 of 0.5 mol dm3 HCl and 25 cm3 of 1 mol dm3 Ba(OH)2

50 cm3 of 1 mol dm3 HCl and 50 cm3 of 1 mol dm3 NH3

Astatine is below iodine in Group VII of the Periodic Table.


Which of the following properties are likely to be true for astatine and its
compounds?
1

Silver astatine is insoluble in concentrated ammonia.

Astatine is a weaker oxidising agent than iodine.

Silver astatide has more covalent character than silver iodide.

Progress of reaction
Which of the following are possible overall equations of the above reaction?
1

J+KL

2J + K L

J + 2K L

37

Which of the following compounds will give a precipitate of triiodomethane


when warmed with iodine and aqueous sodium hydroxide?
1

CH3CH2CH(OH)CH3

HCOCH2I

CH3CH2COOCH(CH3)2

[Turn over
H2 Chemistry 9647/01 NYJC J2/2015 Prelim

H2 Chemistry 9647/01 NYJC J2/2015 Prelim

19
The responses A to D should be selected on the basis of

A
B
C
1, 2 and 3
are
correct

1 and 2
only are
correct

NANYANG JUNIOR COLLEGE


JC 2 PRELIMINARY EXAMINATION
Higher 2

2 and 3
only are
correct

1 only
is
correct

CHEMISTRY

9647/01
29 September 2015

Paper 1 Multiple Choice

No other combination of statements is used as a correct response.

1 hour
Additional Materials:

38

39

C6H5CH2OH and C6H5OH react in a similar way with a reagent W.


What could W be?

READ THESE INSTRUCTIONS FIRST

sodium

propanoic acid

phosphorus pentachloride

There are forty questions on this paper. Answer all questions. For each question there are
four possible answers A, B, C and D.
Choose the one you consider correct and record you choice in soft pencil on the separate
Answer Sheet.

ICl

Each correct answer will score one mark. A mark will not be deducted for a wrong answer.
Any rough working should be done in this booklet.

HCN, NaOH

C
H3C

Read the instructions on the Answer Sheet very carefully.

CH3CH=CHCH3
O

40

Write in soft pencil.


Do not use staples, paper clips, highlighters, glue or correction fluid.
Write your name, class and tutors name on the Answer Sheet in the spaces provided
unless this has been done for you.

In which reaction is a racemic mixture being formed as a product?

Multiple Choice Answer Sheet


Data Booklet

CH3CH2CH2C(CH3)(CH2CH3)Br

NH3

An amino acid has the structural formula NH2COCH2CH(NH2)CO2H. Which of


the following statements apply to this amino acid?
1

It is an amino acid which is optically active.

In a polypeptide, the R group of this amino acid maintains the


secondary structure by forming hydrogen bonds with polar R groups of
other amino acids.

In a solution of pH
NH3+COCH2CH(NH3+)CO2H.

1,

this

amino

H2 Chemistry 9647/01 NYJC J2/2015 Prelim

acid

exists

as

This document consists of 19 printed pages.


[Turn over

3
3

Section A
For each question there are four possible answers, A, B, C and D. Choose the one
you consider to be correct.
1

A sample of the element silicon (Si) was vaporised, ionised and passed
through an electric field. It was observed that a beam of 30Si2+ particles gave
an angle of deflection of +2.

A 20 cm3 sample of methanol is ignited with excess oxygen. The volume of


the residual gas obtained was 80 cm 3. When the residual gas was shaken
with aqueous sodium hydroxide, the volume decrease by 20 cm 3. What is the
initial volume of oxygen present? (All volumes are measured at room
temperature and pressure.)

60 cm3

80 cm3

90 cm3

The use of the Data Booklet is relevant to this question.

30

Si2+

+2

100 cm3

+
Assuming an identical set of experimental conditions, by what angle would the
following sample of arsenic be deflected?

Consider the following half-equations.


Isotope
73

Cr2O72 + 14H+ + 6e 2Cr3+ + 7H2O

As

No. of electrons
36

Fe2+ Fe3+ + e
Angle of deflection for 73As

C2O42 2CO2 + 2e
3

What volume of 0.02 mol dm Na2Cr2O7 is required to oxidise 20 cm of an


acidified solution of 0.03 mol dm3 FeC2O4?
A

5 cm3

10 cm3

15 cm3

+3.2

3.2

+1.2

1.2

30 cm3

H2 Chemistry 9647/01 NYJC J2/2015 Prelim

Which of the following pairs have the same shape?


A

PCl3

BF3

NO2+

HNO2

SnCl2

HCN

SO32-

NH3

H2 Chemistry 9647/01 NYJC J2/2015 Prelim

[Turn over

4
5

Beryllium dichloride, BeCl2, reacts with methylamine, CH3NH2 to form a


compound. Which one of the statements is incorrect?

Given the following reaction and its rate law,


2A + B 2C
Rate = k[A][B]

The compound is formed from 1 mole of BeCl2 and 2 moles of CH3NH2.

The Be-N bond formed is chemically similar to a covalent bond.

which of the following graphs correctly represents the rate law?

The compound is capable of forming only two hydrogen bonds per


molecule.

B
rate

[A]
0.01

[A] = 0.01 mol dm-3


[B] = 0.01 mol dm-3

The beryllium atom in beryllium dichloride is electron deficient.


0.005

0.0025

Flask A contains 1 dm3 of neon at 2 kPa and flask B contains 2 dm3 of argon
at 1 kPa. If the flasks are connected at constant temperature, what is the final
pressure?
A

1.33 kPa

1.5 kPa

1.67 kPa

[A][B]

2 kPa

rate

2x

Time

[C]
0.02
0.015

The use of the Data Booklet is relevant to this question.

[A] = 1.00 mol dm-3


[B] = 0.01 mol dm-3

0.01

Aluminium oxide is a stable ionic compound.


Al (s) Al3+ (g) + 3e
O2 (g) + 2e O2- (g)

H = +5467 kJ mol-1
H = +897 kJ mol-1

[B]

x 2x

Time

Which of the following statements is incorrect?


A

The enthalpy change of atomisation of aluminium is +330 kJ mol-1.

The enthalpy change


13625 kJ mol-1.

The lattice energy of aluminium oxide is more exothermic than the


enthalpy change of formation of aluminium oxide.

The sum of the first and second electron affinities of oxygen is


+649 kJ mol-1.

of

formation

H2 Chemistry 9647/01 NYJC J2/2015 Prelim

of

aluminium

oxide

9
is

The decomposition of a sample of H2O2 solution is a first order reaction with a


rate constant of 7.70 x 10-4 s-1. What is the initial concentration in mol dm-3 of
the H2O2 solution if its concentration has dropped to 0.0315 mol dm -3 in
75 min?
A

0.1091

0.1575

0.5040

H2 Chemistry 9647/01 NYJC J2/2015 Prelim

1.008

[Turn over

6
10

One type of rechargeable battery makes use of the nickel-cadmium cell, in


which nickel and cadmium electrodes, coated with the respective hydroxides,
are immersed in potassium hydroxide solution. During discharge, the
cadmium electrode releases electrons to the external circuit.

13

The titration curve below shows the reaction between a 20.0 cm3 of
0.20 mol dm3 of an acid, against 0.10 mol dm3 of base.
pH

The relevant electrodes are:


Cd(OH)2(s) + 2e Cd(s) + 2OH(aq)
NiO(OH)(s) + H2O(l) + e Ni(OH)2(s) + OH(aq)
Which of the following is incorrect about this cell during discharge?

11

The concentration of the potassium hydroxide solution remains


constant.

The mass of the cadmium electrode decreases as cadmium is used up.

Cadmium is the negative electrode.

The electron flows from cadmium electrode to nickel electrode in the


external circuit.

40
Volume of 0.10 mol dm3 base added / cm3

Which of the following sets of reagents could give this curve?

Which of the following statements about the electrolysis of brine using a


diaphragm cell is correct?

I
II
III
IV

H2SO4 and NaOH


CH3COOH and KOH
4-hydroxybenzoic acid and NaOH
benzoic acid and NaOH

The ratio of chlorine : hydrogen : sodium hydroxide formed is 1:2:1.

I only

Sodium ions migrate through the diaphragm from cathode to anode.

I & III

When the diaphragm is removed, sodium chlorate(V) is formed at room


temperature.

II & IV

II, III & IV

Chlorine is produced at the anode while the pH of the electrolyte


around the cathode increases.
14

12

A sparingly soluble aluminium salt dissociates in aqueous solution according


to the equation given.

At 450 C, 50 % of M decomposes according to the equation:

Al2X3 (s) 2Al3+ (aq) + 3X2- (aq)

M(g) 2N(g) + Q(s)


If the equilibrium pressure is 2 atm, what is the equilibrium constant, Kp for the
reaction at this temperature?
A

0.148

If the solubility product, Ksp, of Al2X3 is S, what is the value of [Al3+ (aq)] at
equilibrium?
1

2.67
A

H2 Chemistry 9647/01 NYJC J2/2015 Prelim

1
2

2 2
S
5

1
5

8 5
S

27

H2 Chemistry 9647/01 NYJC J2/2015 Prelim

[Turn over

8
15

The graph below shows the variation in the enthalpy change of vaporisation,
Hvap for 8 consecutive elements in the Periodic Table, all with atomic
number 20.

17

Hvap / kJ mol1

When crystalline potassium chromate(VI) was dissolved in water, a yellow


solution T was formed. Addition of dilute sulfuric acid to T gave an orange
solution U. When hydrogen sulfide was bubbled through solution U, there was
a color change in the solution and yellow sulfur was produced.
Which process did not occur in the above experiment?

ligand exchange

precipitation

redox reaction

acid-base

H
G
D
C

18

E
F
Atomic number

What can be deduced from the above graph?

16

E is soluble in warm benzene whereas F is not.

The chlorides become less acidic from A to C.

The pH of the solution containing a mixture of oxide of G and oxide of


D is greater than 7.

The oxide of A reacts with excess aqueous sodium hydroxide to form a


soluble complex.

P, Q and R are Group II elements. They form compounds with the following
properties:

P(NO3)2 has a higher thermal decomposition temperature than


R(NO3)2.

RCO3 has a more endothermic


decomposition than QCO3.

enthalpy

change

19

The compound 1,2-diaminoethane, H2NCH2CH2NH2, is a bidentate ligand; in


formulae, it is usually abbreviated to en. When 1,2-diaminoethane is added to
[Co(NH3)6]2+ in aqueous solution, [Co(en)3]2+ is formed. What is the best
explanation for this?
A

There are much stronger bonds between the ligands and the cobalt(II)
ion in [Co(en)3]2+ than in [Co(NH3)6]2+.

When [Co(en)3]2+ is formed from [Co(NH3)6]2+ the reaction is


exothermic.

When [Co(en)3]2+ is formed from [Co(NH3)6]2+ the entropy change is


positive.

When [Co(en)3]2+ is formed from [Co(NH3)6]2+ the reaction has a low


activation energy.

An unknown compound has the following features:

of

Which of the following statements is correct?

It is non-cyclic.
It has a relative molecular mass of x.
It produces a silver mirror with Tollens Reagent.
It reacts with H2 in the presence of Ni to form a product with the
molecular mass of (x + 4).

PO has a more exothermic lattice energy than RO.

QCl2 has more covalent character than PCl2.

R is more reducing than P.

54

Q is Mg and R is Ba.

56

58

60

H2 Chemistry 9647/01 NYJC J2/2015 Prelim

What is the smallest possible value of the molecular mass, x?

H2 Chemistry 9647/01 NYJC J2/2015 Prelim

[Turn over

10

11

20

22
OH
OH

In the free radical substitution of 2,3-dimethylpentane with chlorine, a mixture


of mono-chlorinated compounds were obtained.
How many different structural isomers can be formed?

HO

Q is reacted with acidified hot concentrated manganate(VII) ions.


How many stereoisomers and oxygen atoms will be present in the organic
product(s) formed?

23

A sequence of reactions is shown below.


Br

No. of stereoisomers

No. of oxygen atoms

22

22

23

23

step 1

step 2

O
OH

step 3

Br2 in CCl4
Br
Br

21

Covalent bonds are formed by orbital overlap. The shape of unsaturated


hydrocarbon molecules can be explained in terms of hybridisation of orbitals.

Which of the following shows the correct list of reagents and conditions for the
sequence?

Which bond is not present in HCCCH=C=CHCH2CH3?


A

a bond formed by 2p 2p overlap

a bond formed by 1s 2sp overlap


2

a bond formed by 2sp 2s overlap

a bond formed by 2sp 2sp overlap

step 1

step 2

step 3

AlBr3

NaBH4

Ethanolic NaOH

AlBr3

Na in ethanol

Al2O3

LiAlH4

Acidified K2Cr2O7

conc. H2SO4

LiAlH4

H2 with Ni

Al2O3

H2 Chemistry 9647/01 NYJC J2/2015 Prelim

H2 Chemistry 9647/01 NYJC J2/2015 Prelim

[Turn over

12
24

13

What type of reaction is not involved in the formation of the organic product
below?
O

NCH 2 CH 3

CH3CH2Br

NaOH
O

Which reaction could be used as a possible synthesis of this compound?

N Na

NH

26

KOH
O
O

NH2

NH2

NH2

NH2

CH3COOH

(CH3)2CHBr

(CH3)2CHBr

CH3COOH

CH3COBr

(CH3)2CHBr

(CH3)2CHBr

CH3COBr

CH 3 CH 2 NH 2

redox

neutralisation

hydrolysis

25

nucleophilic substitution

One mole of compound T is warmed with excess ethanolic silver nitrate.


How many moles of silver chloride will be precipitated?

CH 2 Cl

27

In which pair of molecules is the pKb of species I bigger than that of species II ?

Cl
CH=CHCl

II

CH3CH2NH2

CH3CON(CH3)2

B
O

COCl
C

CH3COO

CH2ClCOO

D
A

H2 Chemistry 9647/01 NYJC J2/2015 Prelim

NH2

H3 C

H2 Chemistry 9647/01 NYJC J2/2015 Prelim

NH2

[Turn over

14
28

15

A compound R, C13H13O4Br undergoes the following reactions:

1 mole of R reacts with Tollens Reagent but not Fehlings solution to


produce 4 moles of silver.
R reacts with hot ethanolic sodium hydroxide to form 3 isomeric
products.
R reacts with hot acidified potassium dichromate to form two organic
compounds.

30

Denaturation of proteins occurs due to the disruption of the tertiary and


quaternary structures where various R group interactions are destroyed by
external factors such as pH changes and heavy metal ions.
Which of the following incorrectly shows the R group interactions by such
external factors?

What is the structure of R?

CHO

COOCH 3

COOCH 3

CHO

CHBrCH 2 CH3

hydrogen bonds

heating

van der Waals

extreme pH

disulfide bridges

heavy metal ions

ionic interactions

heavy metal ions

CH2 CHBrCH 3

COCHO

External factor

CHO

CHO

R group interactions

CHO

CH3

OCOCH 3

CHO

CHBrCH 2 CH 3

COOH
CH 2 CHBrCH 3

29
CH 2 CN
COOH
O
HO

O
CH=CH 2

How many moles of HCl(g) are required to react with one mole of the above
molecule?
A

H2 Chemistry 9647/01 NYJC J2/2015 Prelim

H2 Chemistry 9647/01 NYJC J2/2015 Prelim

[Turn over

16

17

Section B
For each of the questions in this section one or more of the three numbered
statements 1 to 3 may be correct.

The responses A to D should be selected on the basis of

A
B
C
1, 2 and 3
are
correct

Decide whether each of the statements is or is not correct (you may find it helpful to
put a tick against the statements which you consider to be correct).
The responses A to D should be selected on the basis of
A
1, 2 and 3
are
correct

B
1 and 2
only are
correct

C
2 and 3
only are
correct

D
1 only
is
correct

34

three elements
the composition, by mass, C, 54.5%; H, 9.1%

What could these molecules be?

32

CH3CH2CH2CO2H

OHCCH2CH2CH2OH

CH3CH=CHCH2SH

1 only
is
correct

In a chemical reaction, J reacts with K to form L. The initial rates of the


reaction are measured in the following experiment.

Three organic molecules each have

2 and 3
only are
correct

No other combination of statements is used as a correct response.

No other combination of statements is used as a correct response.


31

1 and 2
only are
correct

Experiment

[J]/ mol dm3

[K]/ mol dm3

Initial rate/ mol dm3

0.150

0.250

2.80 x 105

0.150

0.500

5.60 x 105

3
4

0.075
0.075

0.500
0.250

2.80 x 105
1.40 x 105

The energy profile diagram for the reaction is as shown:


Energy

When a sample of gas is compressed at constant temperature from 20 atm to


50 atm, its volume changes from 96 cm3 to 46 cm3. Which of the following
statements is a possible explanation of this behaviour?

Progress of reaction
Which of the following are possible overall equations of the above reaction?

33

The gas particles are in a state of continual, random motion.

The gas particles have a finite volume.

There are forces of attraction between gas particles.

J+KL

2J + K L

J + 2K L

Which of the following relationships are correct?


1

2 Hformation(HI) = BE (HH) + BE (II)

2 Hatomisation(I2) = Hsublimation(I2) + BE (II)

Hformation(H2O) = Hcombustion(H2)

[Turn over
H2 Chemistry 9647/01 NYJC J2/2015 Prelim

H2 Chemistry 9647/01 NYJC J2/2015 Prelim

18

19

The responses A to D should be selected on the basis of

A
B
C
1, 2 and 3
are
correct

1 and 2
only are
correct

2 and 3
only are
correct

The responses A to D should be selected on the basis of

A
B
C

D
1 only
is
correct

1, 2 and 3
are
correct

1 and 2
only are
correct

2 and 3
only are
correct

1 only
is
correct

No other combination of statements is used as a correct response.

No other combination of statements is used as a correct response.

35

38

Which of these pairs of substances, when mixed, would lead to the same
temperature rise as the reaction below?
50 cm3 of 1 mol dm3 HCl and 50 cm3 of 1 mol dm3 KOH

36

50 cm of 0.5 mol dm

H2SO4 and 50 cm of 1 mol dm


3

HCl and 25 cm of 1 mol dm

25 cm of 0.5 mol dm

50 cm3 of 1 mol dm3 HCl and 50 cm3 of 1 mol dm3 NH3

C6H5CH2OH and C6H5OH react in a similar way with a reagent W.


What could W be?
1

sodium

propanoic acid

phosphorus pentachloride

KOH

Ba(OH)2
39

In which reaction is a racemic mixture being formed as a product?

Astatine is below iodine in Group VII of the Periodic Table.


Which of the following properties are likely to be true for astatine and its
compounds?

ICl
CH3CH=CHCH3
O

Silver astatine is insoluble in concentrated ammonia.

H3C

37

Astatine is a weaker oxidising agent than iodine.


3

HCN, NaOH

Silver astatide has more covalent character than silver iodide.

Which of the following compounds will give a precipitate of triiodomethane


when warmed with iodine and aqueous sodium hydroxide?
1

CH3CH2CH(OH)CH3

HCOCH2I

CH3CH2COOCH(CH3)2

H2 Chemistry 9647/01 NYJC J2/2015 Prelim

40

CH3CH2CH2C(CH3)(CH2CH3)Br

NH3

An amino acid has the structural formula NH2COCH2CH(NH2)CO2H. Which of


the following statements apply to this amino acid?
1

It is an amino acid which is optically active.

In a polypeptide, the R group of this amino acid maintains the


secondary structure by forming hydrogen bonds with polar R groups of
other amino acids.

In a solution of pH
NH3+COCH2CH(NH3+)CO2H.

1,

this

amino

H2 Chemistry 9647/01 NYJC J2/2015 Prelim

acid

exists

as

20
NANYANG JUNIOR COLLEGE
JC 2 PRELIMINARY EXAMINATION
Higher 2

Answer Key
1
2
3
4
5

C
C
D
D
C

6
7
8
9
10

A
B
D
D
B

11
12
13
14
15

D
D
C
D
D

16
17
18
19
20

B
A
C
B
D

21
22
23
24
25

C
C
B
A
B

26
27
28
29
30

D
D
B
A
B

31
32
33
34
35

A
C
C
C
D

36
37
38
39
40

A
A
D
A
B

CANDIDATE
NAME
TUTORS
NAME

CLASS

CHEMISTRY

9647/02
16 September 2015

Paper 2 Structured Questions

2 hours

Candidates answer on the Question Paper


Additional Materials:

Data Booklet

READ THESE INSTRUCTIONS FIRST


Write your name and class on all the work you hand in.
Write in dark blue or black pen.
You may use an HB pencil for any diagrams, graphs.
Do not use staples, paper clips, glue or correction fluid.
Answer all questions.
The use of an approved scientific calculator is expected, where appropriate.
A Data Booklet is provided.
At the end of the examination, fasten all your work securely together.
The number of marks is given in brackets [ ] at the end of each question or part question.

For Examiners
Use

/12

/24

/10

/10

/16

Total

/72

This document consists of 20 printed pages.


[Turn Over
H2 Chemistry 9647/01 NYJC J2/2015 Prelim

2
Answer all questions in the spaces provided.
1

For
Examiner's
Use

For
Examiner's
Use

(a) Using the information given, prove that a minimum concentration of


5.0 x 10-3 mol dm-3 aqueous hydrochloric acid is required for use in the titration.

Planning (P)
When a solid is dissolved in a solvent, the entropy of the system nearly always
increases. This is mainly due to the increased disorder of the solute particles as the
forces that hold the solid together are overcome.
Entropy change cannot be measured directly in the laboratory. It can, however, be
determined by exploring the relationship between the equilibrium constant and
thermodynamic state properties of the system: Gibbs free energy, enthalpy and
entropy.
The tendency for a reaction to reach an equilibrium is driven by the Gibbs free energy.
The equilibrium constant, K is related to Gibbs free energy change, G by the
relationship:

[1]
(b) Using the information given above, you are required to write a plan to determine
the solubility product of M(OH)2 at various temperatures.

G = R T ln K
where R is the ideal gas constant in units of Jmol-1K-1, T is the temperature in Kelvin,
ln represents a logarithm to the base e.

You may assume that you are provided with:

In turn, the energy behind this driving force comes from the enthalpy and entropy of
reaction in the system, and G has been defined in terms of enthalpy and entropy
changes H and S, at temperature T as:

solid M(OH)2;
0.100 mol dm-3 aqueous hydrochloric acid;
a thermostatic water bath;
the apparatus and chemicals normally found in a college laboratory.

G = H T S
Your plan should include details of:

where T is the temperature in Kelvin.

You are provided with a sample of M(OH)2 where M could be Pb or Ca.


Some information regarding the two hydroxides is given below:
Hydroxide

Mr

Density
/ g cm-3

pH of saturated
solution (at 25 oC)

Pb(OH)2

241.2

7.41

12.1

Ca(OH)2

74.1

2.21

11.7

the preparation of 250 cm3 of 5.0 x 10-3 mol dm-3 aqueous hydrochloric acid;
the preparation of a saturated solution of M(OH)2;
the extraction of samples at various temperatures;
the titration of the samples;
how Ksp would be determined.

You are to determine the entropy change of dissolution of M(OH) 2, Sdissolution, by


determining the solubility product, Ksp at various temperatures. A saturated solution of
M(OH)2 (aq) should be prepared and a 10 cm 3 sample extracted and titrated against
aqueous hydrochloric acid of a suitable concentration. After K sp at various
temperatures have been determined, a suitable straight line graph should then be
plotted to find entropy change of dissolution.

H2 Chemistry 9647/02 NYJC J2/15 Prelim

[Turn Over

H2 Chemistry 9647/02 NYJC J2/15 Prelim

[Turn Over

For
Examiner's
Use

For
Examiner's
Use

[8]

(c) State a set of variables that can be plotted to obtain a straight line graph to
determine Sdissolution of M(OH)2. Suggest how Sdissolution can be determined from
the graph.

A graph of (y-axis) against (x-axis)


can be plotted.

[1]

(d) Describe a simple chemical test to identify the cation present in M(OH)2.

[2]
[Total: 12]

H2 Chemistry 9647/02 NYJC J2/15 Prelim

[Turn Over

The following question is about ammonia and its derivatives.


H2 Chemistry 9647/02 NYJC J2/15 Prelim

[Turn Over

For
Examiner's
Use

Hydrazine is a colourless flammable liquid with an ammonia-like odour. It is highly toxic


and dangerously unstable unless handled in solution. Hydrazine, N 2H4, is a weak base
with the following Kb values.
[Kb1 = 8.5 x 107, Kb2 = 8.9 x 1016]
(a) (i)

(iv)

For
Examiner's
Use

With reference to the procedure and answers in (iii), sketch a graph to show
the changes in pH when the above titration experiment was performed,
labelling the buffer region (if any) and the pH values calculated in (iii).

Write equations to show how hydrazine ionises in water.

(ii)

Hence suggest a reason for the difference in the Kb values.

(iii)

25.0 cm3 of 0.10 mol dm3 hydrazine solution was titrated with
0.20 mol dm-3 HCl solution. 12.50 cm3 of HCl solution was added to reach
the first end point.

[7]
(b)

Determine the pH for the following:


(I) hydrazine solution
(II) solution at equivalence point

Hydrazine can be used as a low-power propellant for the maneuvering thrusters of


spacecraft by producing gas in a reaction with hydrogen peroxide.
N2H4 + 2H2O2 N2 + 4H2O

(Ignore the effect of Kb2 for both calculations.)


(i)

Using bond energies from the data booklet, calculate the enthalpy change of
this reaction.

(ii)

The actual value of the enthalpy change of the above reaction is


789 kJ mol-1. Suggest a reason for the discrepancy.

[3]

H2 Chemistry 9647/02 NYJC J2/15 Prelim

[Turn Over

H2 Chemistry 9647/02 NYJC J2/15 Prelim

[Turn Over

8
(c)

For
Examiner's
Use

The Haber Process manufactures ammonia.


N2(g) + 3H2(g)

For
Examiner's
Use

(ii)

In a 5.0 dm3 cylinder, 8.0 g of ammonia at 0 C exerts a pressure, p.


Calculate the pressure, p.

(iii)

In fact, the pressure inside the cylinder is 1.9 x 10 5 Pa under these


conditions. Explain why this differs from the pressure you calculated in (ii).

H < 0

2NH3(g)

The table below shows the percentage of ammonia by volume in equilibrium


mixtures at various pressures at 400 C. In all cases, N2 and H2 were mixed in a
1:3 molar ratio.
Pressure / atm

%NH3 at 400oC

10

3.85

100

25.1

200

31.4

With reference to the data above, explain how changes in pressure affect the
percentage of NH3 in the equilibrium mixture.

[5]
(e)

The diagram below shows how the entropy of ammonia varies with temperature
at a pressure of 100 kPa. In this diagram, ammonia is a solid at point A and a
gas at point F.

[1]
(d)

(i)

S / JK mol

Using the ideal gas equation, sketch two graphs below at 0 C and 100 C
respectively for a fixed mass of gas. Label your graphs clearly.
V / dm

A
0

(i)

B
T

Temperature/K

State the significance of the entropy value for ammonia at 0 K.

(ii)

1
/ kPa1
p

H2 Chemistry 9647/02 NYJC J2/15 Prelim

Temperature T is marked on the diagram. What does the value of this


temperature represent?

[Turn Over

H2 Chemistry 9647/02 NYJC J2/15 Prelim

[Turn Over

10
(iii)

11

For
Examiner's
Use

By considering the change in entropy values, explain the shape of the


graph from D to F.

(f)

For
Examiner's
Use

Ammonia borane (or borazane) is the chemical compound with the formula
BH3NH3. It is formed when BH3 accepts an electron pair from NH3, and is
commonly known as an adduct.

D to E :

BH3NH3 and NH3 differ markedly in their physical states. Explain this in terms of
their structure and bonding.

Compound

Physical state at room temperature

BH3NH3

solid

NH3

gas

E to F:

[4]

Diborane, B2H6 can also accept electron pairs. B2H6 reacts with excess ammonia
to form B2H62NH3. The compound B2H62NH3 is ionic, and comprises of one
singly charged cation and one singly charged anion. Given that the anion is
[BH4], suggest a formula and draw the dot-and-cross diagram for the cation.
(i)

cation:

[4]
[Total: 24]

H2 Chemistry 9647/02 NYJC J2/15 Prelim

[Turn Over

H2 Chemistry 9647/02 NYJC J2/15 Prelim

[Turn Over

12
3

13

For
Examiner's
Use

Cobalt forms many complexes with ligands such as NH3 and Cl.

(e)

For
Examiner's
Use

State the formula of the cobalt-containing complex in Y. Suggest the sign of


Sformation of Y and explain your reasoning.
Formula: ..
..

Na2CO3(aq)

..

excess
CoCl2(aq)

NH3(aq)

pink solution

H2O2(aq)

..

pale brown

dark brown

solution

solution

concentrated

excess

HCl

H2NCH2CH2NH2

Z
(a)

[2]
(f)

Identify the cobalt-containing complex in solution Z and suggest why the


coordination number is different from that identified in (a).
..
..

..

Suggest the formula and shape of the complex responsible for the pink colour in
aqueous CoCl2.

[2]
[Total:10]

Formula: ..
Shape:
(b)

..

[1]

Suggest the formula for V and state its appearance.


Formula: ..
Appearance: ...

(c)

[2]

Write an equation for the reaction when the pink solution was converted to W.

(d)

[1]

Suggest the formula for the cobalt-containing complex in X and state the role of
H2O2 in this reaction.
Formula: ..
Role:

...

H2 Chemistry 9647/02 NYJC J2/15 Prelim

[2]

[Turn Over

H2 Chemistry 9647/02 NYJC J2/15 Prelim

[Turn Over

14
4

15

For
Examiner's
Use

The structure shown below is the effective ingredient of vanilla.

(c)

For
Examiner's
Use

Explain the roles of ether and anhydrous magnesium sulfate in Step IV.

HO

....
....

O
O

....

Vanillin

[2]
Vanillin was subjected to the following steps:
(d)
I

Vanillin was added into a flask containing an ethanolic solution of NaBH 4,


shaking and cooling the flask well to keep the temperature below 30oC.

Suggest another reagent and condition that could be used in place of ethanolic
NaBH4 to bring about the same reaction and write a balanced equation for the
reaction.

II

The flask was then cooled at room temperature for about 20 min.

III

6 mol dm-3 hydrochloric acid was then added into the reaction mixture to
hydrolyse the excess NaBH4 and to liberate the alcohol.

....

IV

Ether was added to the reaction mixture and shaken. The ether layer was
separated and then mixed with anhydrous magnesium sulfate.

....

(a)

....

The mixture was then filtered and the ether solution was evaporated over a hot
water bath to obtain the product.

[2]

Why do you think the temperature of the flask was kept below 30 C?

....
[1]

(b)

Suggest why NaBH4 was introduced as an ethanolic solution?

....
....
....
[2]

H2 Chemistry 9647/02 NYJC J2/15 Prelim

[Turn Over

H2 Chemistry 9647/02 NYJC J2/15 Prelim

[Turn Over

16
(e)

17

For
Examiner's
Use

The reduction of carbonyl compounds by metal hydrides such as sodium


borohydride, NaBH4, forms alcohols. The attacking species is the BH4 ion
which, in effect, is a carrier of hydride ions, H. The reduction mechanism
involves two steps.

(a)

For
Examiner's
Use

Chloroprocaine is an ester with the following structure. It is commonly used as


anesthetic during surgery.
CH3
N

Step 1:
Attack of the hydride ion, H, at the carbonyl carbon gives a tetrahedral
intermediate anion.

O
H2N

CH3

Step 2:
The anion reacts with H2O to give the alcohol product.

Cl

Suggest the name of this mechanism. Hence, draw the mechanism of the
reduction of Vanillin. In your answer, show any relevant charges, dipoles or lone
pairs of electrons you consider important in this mechanism.

Chloroprocaine

(i)

State the type and shape of hybrid orbitals at nitrogen atoms labelled
N1 and N2.

Name of mechanism: ....


Hybrid orbital

Step 1:

N1

N2

Type

Shape

(ii)

With reference to the answer in (i), State and explain which nitrogen atom
is the weaker base.

Step 2:

....
....
....

[4]
[3]
[Total: 10]

H2 Chemistry 9647/02 NYJC J2/15 Prelim

[Turn Over

H2 Chemistry 9647/02 NYJC J2/15 Prelim

[Turn Over

18
(b)

19

For
Examiner's
Use

One suggested scheme of synthesis for chloroprocaine is outlined below:


KMnO4/H+

CH3

For
Examiner's
Use

replacement of chloroprocaine. Suggest a distinguishing test to


differentiate between the two compounds.
CO 2H

CH3

reflux
H2N

O
H2N

step I

OH

OH

CH3
NH 2

Cl
PCl5

Compound X

step II
H3C

CH3

Test: ....
N

N
O

H2N

HO

Cl

CH3

COCl

CH3

step III

H2N

Observation: ..
..

Cl

[6]

chloroprocaine

(i)

The synthesis suggested is flawed. By considering the reagents and


conditions used for each step, state and explain the steps that are
incorrect.

(c)

Chloroprocaine generally function by fitting into a receptor site of protein


molecules in the body. A simplified diagram of the receptor site is shown below.

ser

....
cys

....

CH2

CH2OH

....
CH3

cys
CH2

(ii)

A newly synthesised compound X has been suggested as potential


H2 Chemistry 9647/02 NYJC J2/15 Prelim

[Turn Over

CH3

(i)

CH

val

State two different types of interactions that can be formed between a


H2 Chemistry 9647/02 NYJC J2/15 Prelim

[Turn Over

20

For
Examiner's
Use

chloroprocaine molecule and the receptor protein. Draw diagrams in the


boxes below to show how these interactions are formed.
Type of interaction:

Type of interaction:

NANYANG JUNIOR COLLEGE


JC 2 PRELIMINARY EXAMINATION
Higher 2
CANDIDATE
NAME
TUTORS
NAME

CLASS

CHEMISTRY

9647/02

Paper 2 Structured

16 Sept 2015
2 hours

Candidates answer on the Question Paper


Additional Materials:

Data Booklet

READ THESE INSTRUCTIONS FIRST

(ii)

Protein molecules undergo denaturation due to various factors, resulting


in loss of biological functions. What is meant by the term denaturation?
....
....

Write your name and class on all the work you hand in.
Write in dark blue or black pen.
You may use an HB pencil for any diagrams, graphs.
Do not use staples, paper clips, glue or correction fluid.
Answer all questions.
The use of an approved scientific calculator is expected, where appropriate.
A Data Booklet is provided.
At the end of the examination, fasten all your work securely together.
The number of marks is given in brackets [ ] at the end of each question or part question.

(iii)

Copper(I) ions, Cu+ (aq), are highly toxic to humans due to their effects on
proteins. With reference to the receptor site shown, suggest how
copper(I) ions may affect the proteins ability to act as receptor site.

For Examiners
Use

....

/12

....

/24

/10

/10

/16

Total

/72

[6]
[Total: 16]

This document consists of 16 printed pages.


[Turn Over
H2 Chemistry 9647/02 NYJC J2/15 Prelim

[Turn Over

For
Examiner's
Use

For
Examiner's
Use

Answer all questions in the spaces provided.


(a)
1

Planning (P)

Using the information given, prove that a minimum concentration of


5.0 x 10-3 mol dm-3 aqueous hydrochloric acid is required for use in the titration.

When a solid is dissolved in a solvent, the entropy of the system nearly always
increases. This is mainly due to the increased disorder of the solute particles as the
forces which hold the solid together are overcome.

To find the minimum concentration required for complete reaction, we will


assume the more alkaline solution Pb(OH)2 is present.
pOH = 14 12.1 = 1.9

Entropy change cannot be measured directly in the laboratory. It can, however, be


determined by exploring the relationship between the equilibrium constant and
thermodynamic state properties of the system: Gibbs free energy, enthalpy and
entropy.
The tendency for a reaction to reach an equilibrium is driven by the Gibbs free energy.
The equilibrium constant, K is related to Gibbs free energy change, G by the
relationship:
G = R T ln K
where R is the ideal gas constant in units of Jmol-1K-1, T is the temperature in Kelvin,
ln represents a logarithm to the base e.
In turn, the energy behind this driving force comes from the enthalpy and entropy of
reaction in the system, and G has been defined in terms of enthalpy and entropy
changes H and S, at temperature T as:
G = H T S

For a 10.0 cm3 pipetted sample, assuming titre volume (of HCl) is 25 cm3.
10-1.9 x 10/1000 = [HCl] x 25/1000
[HCl] = 5.0 x 10-3 mol dm-3 (shown) [1]

[1]
(b)

Using the information given above, you are required to write a plan to determine
the solubility product of M(OH)2 at various temperatures.
You may assume that you are provided with:

solid M(OH)2;
0.100 mol dm-3 aqueous hydrochloric acid;
a thermostatic water bath;
the apparatus and chemicals normally found in a college laboratory.

Your plan should include details of:

where T is the temperature in Kelvin.


You are provided with a sample of M(OH)2 where M could be Pb or Ca. Some
information regarding the two hydroxides are provided below:
Hydroxide

Mr

Density
/ g cm-3

pH of saturated
solution (at 25 oC)

Pb(OH)2

241.2

7.41

12.1

Ca(OH)2

74.1

2.21

11.7

the preparation of 250 cm3 of 5.0 x 10-3 mol dm-3 aqueous hydrochloric acid;
the preparation of a saturated solution of M(OH)2;
the extraction of samples at various temperatures;
the titration of the samples;
how Ksp would be determined.

Justification
V (0.100) = 5.0 x 10-3 (250)
V = 12.5 cm3

You are to determine the entropy change of dissolution of M(OH) 2, Sdissolution, by


determining the solubility product, Ksp at various temperatures. A saturated solution of
M(OH)2 (aq) should be prepared and a 10 cm 3 sample extracted and titrated against
aqueous hydrochloric acid of a suitable concentration. After K sp at various
temperatures have been determined, a suitable straight line graph should then be
plotted to find entropy change of dissolution.

H2 Chemistry 9647/02 NYJC J2/15 Prelim

[Turn Over

Preparation of 5.0 x 10-3 mol dm-3 HCl


1. From a burette, add 12.50 cm3 of 0.100 moldm-3 aqueous HCl into a clean
250 cm3 volumetric flask. (burette + 50.00 cm3 [1])
2. Top up to mark with deionised water. Cap and shake the volumetric flask to
obtain a homogeneous solution. Label the solution FA1. (volumetric flask,
+ procedure [1])

H2 Chemistry 9647/02 NYJC J2/15 Prelim

[Turn Over

4
Preparation of saturated solution of M(OH)2
3. Add about 200 cm3 deionised water in a 250 cm3 beaker and place in a
thermostatic water bath with temperature 30 oC.
4. Monitor the temperature of the water with a 0.1 oC thermometer until the
temperature equals that of the water bath.
5. Stir and add solid M(OH)2 to the beaker until no more solid dissolves.
(Steps 3 & 5 [1])
6. Record the temperature with a 0.1 oC thermometer.
(Steps 4 and 6 [1])
7. Quickly decant/filter about 50 cm3 of solution from the 250 cm3 beaker into a
clean 100 cm3 beaker.
8. Pipette 10.0 cm3 of solution/filtrate into a clean conical flask. Label this
sample 30.0 oC. (steps 7 & 8 [1])
9. Repeat steps 4, 6-8 at four additional temperatures at approximately 35 oC,
40 oC, 45 oC and 50 oC. Stir the mixture in the beaker well between
experiments. Record the temperatures in step 6 in table 1 below.
(3 to 5 experiments at appropriate temperatures to be conducted [1])
Expt
Room temp
35
40
45
45

Temp / oC

For
Examiner's
Use

(c)

For
Examiner's
Use

State a set of variables that can be plotted to obtain a straight line graph to
determine Sdissolution of M(OH)2. Suggest how Sdissolution can be determined
from the graph.
A graph of (y-axis) against (x-axis)
can be plotted.
RT ln Ksp = H T S
A graph of RT ln Ksp / Jmol-1 against T / K can be plotted. S = grad
OR
RT ln Ksp = T S H
A graph of RT ln Ksp / Jmol-1 against T / K can be plotted. S = grad
OR
ln Ksp = S / R (H/R)(1/T)
A graph of ln Ksp against 1/T can be plotted. S = y-intersect x R [1]
[1]

Vol of NaOH required in titration / cm3


(d)

Describe a simple chemical test to identify the cation present in M(OH)2.


To about 1 cm3 of saturated solution of M(OH)2 in a test-tube, add NaOH(aq)
dropwise until in excess.
If M = Pb : white ppt formed, soluble in excess NaOH(aq)
If M = Ca: no ppt formed/white ppt formed, insoluble in excess NaOH(aq)

Table 1
Titration
10. Fill another burette with FA1.
11. Add 2-3 drops of phenolthalein to the conical flask labelled 30 oC.
12. Titrate the OH- with FA1 until a permanent pink is observed with 1 drop
excess FA1. (steps 11 &12 [1])
13. Repeat steps 11 and 12 for the other samples and record the titre volume in
table 1 above.

OR To about 1 cm3 of saturated solution of M(OH)2 in a test-tube, add NH3(aq)


dropwise until in excess.
If M = Pb : white ppt formed (insoluble in excess NaOH (aq))
If M = Ca: no ppt formed
[1] test

[1] observation (for both cations)


[2]
[Total: 12]

Determination of Ksp
Let titre volume be V cm3.
[OH-] = V (3.5x10-3) / 10 = 3.5 x 10-4 V mol dm-3
[M2+] = [OH-] = 1.75 x 10-4 V mol dm-3
Ksp = [M2+][OH-]2 = 2.14 x 10-11 V3 mol3 dm-9 [1]

[8]

H2 Chemistry 9647/02 NYJC J2/15 Prelim

[Turn Over

H2 Chemistry 9647/02 NYJC J2/15 Prelim

[Turn Over

For
Examiner's
Use

The following question is about ammonia and its derivatives.

(iv)

Hydrazine is a colourless flammable liquid with an ammonia-like odour. It is highly


toxic and dangerously unstable unless handled in solution. Hydrazine, N 2H4, is a
weak base with the following Kb values.
[Kb1 = 8.5 x 107, Kb2 = 8.9 x 1016]
(a)

(i)

Write equations to show how hydrazine ionises in water.

(ii)

N2H4 (aq) + H2O (l)


N2H5+ (aq) + OH (aq)
+
N2H5 (aq) + H2O (l)
N2H62+ (aq) + OH (aq)
[1] for both equations
Hence suggest a reason for the difference in the Kb values.

10.5
Buffer region

4.55

25.0 cm3 of 0.10 mol dm3 hydrazine solution was titrated with
0.20 mol dm-3 HCl solution until 12.50 cm3 of HCl solution was added to
reach the first end point.

0.00

12.50

Volume of B/ cm3

[1] correct axes, labels and graph shape


[1] buffer region correctly labelled

Determine the pH for the following:


(1) hydrazine solution
(2) solution at equivalence point

[7]
(b)

(Ignore the effect of Kb2 for both calculations.)

Hydrazine can be used as a low-power propellant for the maneuvering thrusters


of spacecraft by producing gas in a reaction with hydrogen peroxide.

Initial solution in conical flask contains only hydrazine solution.


[OH] = Kb1 (0.10) = 2.915 x 104 mol dm3

N2H4 + 2H2O2 N2 + 4H2O

pOH = lg (2.915 x 104) = 3.54


pH = 14 3.54 = 10.46
the pH for initial solution in the conical flask is 10.5 [1]

(i)

n(N2H5+ (aq)) = 0.0025 mol


Total volume = 25.0 + 12.5 = 37.5 cm 3
0.0025
[N2H5+ (aq)] =
= 0.06667
37.5 1000
1014
1.1108 mol dm3 [1]
Ka1 =
7
8.5 10
+
[H ] = Ka1 (0.06667) 2.80 105 mol dm3
pH = lg [H+] = 4.55
the pH for equivalence point for this titration 4.55 [1]

[Turn Over

Using bond energies from the data booklet, calculate the enthalpy change
of this reaction.
Hrxn = 4(N-H) + 1(N-N) + 4 (O-H) + 2 (O-O) 1(NN) 8(O-H)
= 4(390) + 1(160) + 4(460) + 2(150) 1(994) 8(460) [1]
= 814 kJ mol-1 [1]

N2H4 (aq) + HCl (aq) N2H5+ (aq) + Cl (aq)


Salt hydrolysis: N2H5+ (aq) + H2O (l)
N2H42+ (aq) + H3O+ (aq)

H2 Chemistry 9647/02 NYJC J2/15 Prelim

With reference to the procedure and answers in (iii), sketch a graph to


show the changes in pH when the above titration experiment was
performed, labelling the buffer region (if any) and the pH values
calculated in (iii).
pH

Kb2 is much smaller than Kb1 due to repulsion when a positively charged
N2H5+ gains a positively charged H+.
OR
Kb2 is much smaller than Kb1 due to repulsion between two positively
charged particles. [1]
(iii)

For
Examiner's
Use

(ii)

The actual value of the enthalpy change of the above reaction is


789 kJ mol-1. Suggest a reason for the discrepancy.
The calculated value did not account for the energy absorbed (Hvap of
hydrazine and Hvap of hydrogen peroxide) and energy given out for
H2O (g) H2O (l).
OR
Reactants are not in gaseous state. [1]
[3]

H2 Chemistry 9647/02 NYJC J2/15 Prelim

[Turn Over

(c)

For
Examiner's
Use

The following data refer to the Haber Process for the manufacture of ammonia.
N2(g) + 3H2(g)

Answer:
V / dm3

H < 0

2NH3(g)

The table below shows the percentage of ammonia by volume in equilibrium


mixtures at various pressures at 400 C. In all cases, N2 and H2 were mixed in a
1:3 molar ratio.
Pressure / atm

%NH3 at 400oC

10

3.85

100

25.1

200

31.4

For
Examiner's
Use

100C

0C

straight line

With reference to the data above, explain how changes in pressure affect the
percentage of NH3 in the equilibrium mixture.

1/p / kPa-1

An increase in pressure favours the reaction that produces less number of


moles of gas.
forward reaction produces less number of moles of gas,
increase in pressure results in an increase in the % of NH3 [1]
(d)

(i)

(ii)

[2]
In a 5.0 dm3 cylinder, 8.0 g of ammonia at 0C exerts a pressure, p.
Calculate the pressure, p.
Using pV = nRT
p x 5.0 x 10-3 = (8.0/17) x 8.31 x 273 [1]
p = 213 518 Pa = 213 kPa [1]

Using the ideal gas equation, sketch two graphs in the axis below
at 0 C and 100C respectively for a fixed mass of gas. Label your
graphs clearly. [2]

(iii)

V / dm3

(e)

In fact, the pressure inside the cylinder is 1.9 x 10 5 Pa under these


conditions. Explain why this differs from the pressure you calculated in
(ii).
This is due to the presence of hydrogen bonding between the NH3
molecules, resulting in a lower pressure than expected. [1]
[5]

The diagram below shows how the entropy of ammonia varies with temperature
at a pressure of 100 kPa. In this diagram, ammonia is a solid at point A and a
gas at point F.

S /J K-1mol-1

1/p / kPa1
D

C
A
0

H2 Chemistry 9647/02 NYJC J2/15 Prelim

[Turn Over

B
T

Temperature/K

H2 Chemistry 9647/02 NYJC J2/15 Prelim

[Turn Over

10
(i)

[4]
[Total: 24]
3

Cobalt forms many complexes with ligands such as NH3 and Cl.

Temperature T is marked on the diagram. What does the value of this


temperature represent?

boiling point of ammonia. [1]


(iii)

Na2CO3(aq)

By considering the change in entropy values, explain the shape of the


graph from D to F.

excess
D to E : (Energy is used to break the hydrogen bonds between ammonia
particles hence) There is a change from the liquid to gaseous state.
(Gaseous particles have more disorder/move about more freely) hence
more ways to arrange the particles and there is a large increase in
disorder. [1]

(f)

For
Examiner's
Use

State the significance of the entropy value for ammonia at 0 K.


There is no disorder of the ammonia molecules. [1]

(ii)

11

For
Examiner's
Use

CoCl2(aq)
pink solution

NH3(aq)

H2O2(aq)

pale brown

dark brown

solution

solution

E to F: there is an increase in kinetic energy as temperature increase.

concentrated

excess

There are more ways to distribute the energy among the particles, giving
rise to greater disorder, and entropy of the system increases. [1]

HCl

H2NCH2CH2NH2

[4]
Ammonia borane (or borazane) is the chemical compound with the formula
BH3 NH3. It is formed when BH3 accepts an electron pair from NH3, and is
commonly known as an adduct.

(a)

Suggest the formula and shape of the complex responsible for the pink colour in
aqueous CoCl2.
[1]
[Co(H2O)6]2+, octahedral [1]

BH3 NH3 and NH3 differ markedly in their physical states. Explain this in terms of
their structure and bonding.

(b)

Suggest the formula for V and state its appearance.

[2]

CoCO3 [1]
Purple precipitate/solid (allow pink precipitate/solid) [1]

compound Physical state at room temperature


BH3 NH3
solid
NH3
gas

(c)

Both compounds have simple molecular structure. There are hydrogen bonds
between NH3 molecules and dispersion forces between BH3 NH3 molecules.
Due to the large size of the electron cloud of the BH3 NH3 molecule (which is
more easily polarised), the dispersion forces are much stronger than the
hydrogen bonds between NH3 molecules.
More energy is needed to break the stronger dispersion forces, resulting in BH3
NH3 having a higher melting point, and existing as a solid. NH3 has a lower
boiling point hence exists as a gas.
[1] for structure and bonding
[1] for explaining difference in physical state
Diborane, B2H6 can also accept electron pairs. B2H6 reacts with excess
ammonia to form B2H62NH3. The compound B2H62NH3 is ionic, and comprises
of one singly charged cation and one singly charged anion. Given that the anion
is [BH4]-, suggest the formula and draw a dot-and-cross diagram for the cation.
cation: [BH2(NH3)2]+ [1]
(i)

Write an equation for the reaction when the pink solution was converted to W. [1]
[Co(H2O)6]2+ + 6 NH3 [Co(NH3)6]2+ + 6H2O [1]
Accept [Co(NH3)4(H2O)2]2+

(d)

Suggest the formula for the cobalt-containing complex in X and state the role of
H2O2 in this reaction.
[2]
[Co(NH3)6]3+ [1]
Oxidising agent [1]
Solution formed hence Co (s) cannot be formed

(e)

State the formula of the cobalt-containing complex in Y. Suggest the sign of


Sformation of Y and explain your reasoning.
[2]
[Co(H2NCH2CH2NH2)3]2+ [1]
S would be positive as 4 mol of reactants form 7 mol of products/ there is an
increase in number of moles of particles [1]

[1]
H2 Chemistry 9647/02 NYJC J2/15 Prelim

[Turn Over

H2 Chemistry 9647/02 NYJC J2/15 Prelim

[Turn Over

12

(f)

13

For
Examiner's
Use

Identify the cobalt-containing complex in solution Z and suggest why the


coordination number is different from that identified in (a).
[2]

(d)

[CoCl4]2- [1]
The Cl- ligand is bigger than H2O, hence only 4 Cl- can surround Co2+ [1]
Do not accept if student just states steric hindrance
[Total:10]
The structure shown below is the effective ingredient of vanilla.

For
Examiner's
Use

Suggest another reagent and condition that could be used in place of ethanolic
NaBH4 to bring about the same reaction and write a balanced equation for the
reaction.
[2]
Either: LiAlH4 in dry ether, Room temperature
Or: H2(g), Ni, Heat
HO

HO
+ 2[H]
O

HO

(e)
O
O

Vanillin
Vanillin was subjected to the following steps:
I

OH

Vanillin was added into a flask containing an ethanolic solution of NaBH 4,


shaking and cooling the flask well to keep the temperature below 30oC.

The reduction of carbonyl compounds by metal hydrides such as sodium


borohydride, NaBH4, forms alcohols. The attacking species is the BH4 ion
which, in effect, is a carrier of hydride ions, H. The reduction mechanism
involves two steps.
Step 1:
Attack of the hydride ion, H, at the carbonyl carbon gives a tetrahedral
intermediate anion.
Step 2:
The anion reacts with H2O to give the alcohol product.

II

The flask was then cooled at room temperature for about 20 min.

III

6 mol dm-3 hydrochloric acid was then added into the reaction mixture to
hydrolyse the excess NaBH4 and to liberate the alcohol.

Suggest the name of this mechanism. Hence, draw the mechanism of the
reduction of Vanillin. In your answer show any relevant charges, dipoles or lone
pairs of electrons you consider important in this mechanism.
[3]

IV

Ether was added to the reaction mixture and shaken. The ether layer was
separated and then mixed with anhydrous magnesium sulfate.

Name of mechanism: Nucleophilic addition

The mixture was then filtered and the ether solution was evaporated over a hot
water bath to obtain the product.

(a)

HO

HO

Why do you think the temperature of the flask was kept below 30 oC?
Reaction is vigorously exothermic and hence temperature control is necessary
so that the reaction mixture does not boil over and prevent loss of substances
through volatilization.
[1]

(b)

Step 1:

Suggest why NaBH4 was introduced as an ethanolic solution?

[2]

O
H

O
H

Step 2:
HO

HO

H
O
O

Explain the roles of ether and anhydrous magnesium sulfate in step IV.

NaBH4 is an ionic solid and hence will not mix and react well with vanillin.
Hence, a polar solvent, such as ethanol is used to dissolve the reactants.
(c)

[2]

H
O

[Turn Over

O
+ OH

Ether is used to extract the reduced product.


Anhydrous magnesium sulfate is used to remove any moisture/water from the
product (dry the product).
H2 Chemistry 9647/02 NYJC J2/15 Prelim

OH

[Total: 10]

H2 Chemistry 9647/02 NYJC J2/15 Prelim

[Turn Over

14

(a)

15

For
Examiner's
Use

Chloroprocaine is an ester with the following structure. It is commonly used as


anesthetic during surgery.

(b)

One suggested scheme of synthesis for chloroprocaine is outlined below:

O
H2N

KMnO4/H+

CH3

CH3
N

For
Examiner's
Use

reflux
H2N

CH3

CO 2H

H2N

step I

OH

OH

O
Cl

PCl5

Chloroprocaine

step II
H3C

(i)

State the type and shape of hybrid orbitals at nitrogen atoms labelled N1
and N2. [2]

CH3

N
O

Hybrid orbital

H2N

HO

Cl

CH3

COCl

CH3

step III

H2N

Cl

chloroprocaine

(i)

Type

sp3

sp2

Shape

Tetrahedral

trigonal planar

Step I. [1m for identifying the step]Phenylamine is basic in nature,


hence it will react with acidifying agent to form a salt as the product of
step I. [1m for explanation]

4 correct 2m, 2 correct (must be corresponding pair) 1m


(ii)

The synthesis suggested is flawed. By considering the reagent and


condition used for each step, state and explain the steps which are
incorrect.

With reference to the answer in (i), State and explain which nitrogen atom
is the weaker base
N2 is the weaker base. [1]

Step II. [1m for identifying the step] In the phenol functional group, the
p orbital of oxygen atom can overlap with pi electron cloud of benzene,
strengthening the C-O bond. Hence phenol do not undergo substitution
reaction with PCl5 to form chlorobenzene. [1m for explanation]
Comments: Well done.
Students who didnt get the full credit is because they missed out the
protonation for step I. Or that the explanation on why C-Cl bond is strong
was not clearly conveyed.

Lone pair of electrons residing in unhybridised p orbital on N is


delocalized into the ring, hence less available for donation to acid. [1]
[4]

H2 Chemistry 9647/02 NYJC J2/15 Prelim

[Turn Over

H2 Chemistry 9647/02 NYJC J2/15 Prelim

[Turn Over

16

(ii)

17

For
Examiner's
Use

chloroprocaine molecule and the receptor protein. Draw diagrams in the


boxes below to show how these interactions are formed.

A newly synthesized compound X has been suggested as potential


replacement of chloroprocaine. Suggest a distinguishing test to
differentiate between the two compounds.

Type of interaction:
Hydrogen bonding

CH3
O

For
Examiner's
Use

Type of interaction:
Van der Waals force
1m for both interactions
correct

CH3
NH 2

Cl

Compound X
Test: KMnO4 (aq), H2SO4 (aq), heat [1]
OR K2Cr2O7 (aq), H2SO4 (aq), heat
OR H2SO4 (aq), heat

[1]

Observation:
purple KMnO4 decolourise in chloroprocaine. Purple KMnO4 remains
unchanged / does not decolourise in compound X.
OR orange K2Cr2O7 turns green for chloroprocaine. orange KMnO4
remains unchanged in compound X

[1]

(ii)

OR white ppt appears for chloroprocaine. No ppt for compound X

It is the irreversible process whereby the protein loses its biological


activity due to destruction of R-group interactions. Only the secondary,
tertiary and quaternary structures are disrupted, the primary structure
remains intact. [1]

Comments: Not well done.


Quite a large number wrote the type of reaction instead of the reagents
required.
Adding neutral FeCl3 to the acidic hydrolysed products will not get any
credit because it would have altered the pH of FeCl3.
(c)

[6]
Chloroprocaine generally function by fitting into a receptor site of protein
molecules in the body. A simplified diagram of the receptor site is shown below.

(iii)

Copper(I) ions, Cu+ (aq), are highly toxic to humans due to their effects on
proteins. With reference to the receptor site shown, suggest how
copper(I) ions may affect the proteins ability to act as receptor site.
Cu+ act as reducing agent to denature protein by breaking the disulfide
bond[1], hence breaking the tertiary structure of the protein and result in
its loss of ability to act as receptor site. [1]

ser
CH2

Protein molecules undergo denaturation due to various factors, resulting


in loss of biological functions. What is meant by the term denaturation?

CH2OH

[6]
[Total: 16]

cys S
CH3

S
cys

(i)

CH3
CH2

CH

val

State two different types of interactions that can be formed between a


H2 Chemistry 9647/02 NYJC J2/15 Prelim

[Turn Over

H2 Chemistry 9647/02 NYJC J2/15 Prelim

[Turn Over

2
1

NANYANG JUNIOR COLLEGE


JC 2 PRELIMINARY EXAMINATION
Higher 2

Functional groups in a molecule must be protected in order to react in a desired way with
reagents used in a synthesis. For instance, alcohols undergo oxidation or nucleophilic
substitution reactions readily. To avoid these reactions, the alcohol can be protected by
converting it to an unreactive methoxymethyl ether (MOM), RO

CHEMISTRY

9647/03
21 September 2015

Paper 3 Free Response

methylchloromethyl ether (MOMCl), Cl

2 hours

MOMCl

base
RO

ROH
Step I

Candidates answer on separate paper.


Answer Paper
Data Booklet
Graph Paper

OCH3 .

Protection Process

Candidates answer Section A on the Question Paper

Additional Materials:

OCH3 , using

RO
Step II

OCH3
MOM

Deprotection Process

H+(aq)

READ THESE INSTRUCTIONS FIRST

RO

Write your name and class on all the work you hand in.
Write in dark blue or black pen on both sides of the paper.
You may use a soft pencil for any diagrams, graphs or rough working.
Do not use staples, paper clips, highlighters, glue or correction fluid.

OCH3
MOM

(a)

Answer any four questions.


A Data Booklet is provided.
The use of an approved scientific calculator is expected, where appropriate.
You are reminded of the need for good English and clear presentation in your answers.

H
C OCH3
ROH +
H
intermediate A

H2O

H
C

O + CH3OH

(i) Suggest why ROH is converted to RO in Step I.


(ii) Brnsted-Lowry bases are usually used in Step I of the protection process.
Bulky organic bases such as N,N-diisopropylethylamine (DIPEA) is one
example.

At the end of the examination, fasten all your work securely together.
The number of marks is given in brackets [ ] at the end of each question or part question.

N
For Examiners Use

DIPEA

Section A

Bulky organic bases such as DIPEA are preferred over


less bulky organic bases such as primary amine and
alkalis such as sodium hydroxide.

1
2
3

Explain why primary amine is unsuitable to be used as a base in step I of the


protection process.

(iii) Explain why Step I does not occur if the base was changed to sodium
hydroxide.

(iv) Describe the mechanism of Step II for the protection process using MOMCl.
This document consists of 11 printed pages

(v) State the oxidation state of carbon labelled in intermediate A.

[Turn over

H2 Chemistry 9647/03 NYJC J2/2015 Prelim

Compound F can be made from compound B in four steps, where the first step
involves the protection of alcohol using MOMCl.
COOH
Step III
OH
B

Step IV

MOMCl,
DIPEA

Step V

Acrylic acid is used in the manufacture of a large variety of plastics, paint formulations, and
polymer coatings. More than 1000 kilotonnes of acrylic acid are produced for industrial use
in a year.
O

Step VI

KCN, CH3CH2OH
heat under reflux

OH
F

OH
acrylic acid

(vi) Suggest reagents and conditions for Steps IV and VI, and the structures of
compounds C, D and E.
(vii) State the types of reaction for Steps IV, V and VI.

(a)

[15]

(b)
o

Hf CH3OH(l)
o

393.5 kJ mol

285.8 kJ mol1

Hf CO2(g)
Hf H2O(l)
o

238.7 kJ mol1

Sc CH3OH(l)

from

(ii) Explain one disadvantage of this method of synthesis.

(b) Methanol is currently studied as a potential green fuel that greatly reduces the
pollution to the environment.
2CH3OH(l) + 3O2(g) 2CO2(g) + 4H2O(l)

(i) Suggest a 3-step synthesis of acrylic acid starting


State all reagents and conditions and draw all intermediates.

propene.

[4]

Another method of synthesising acrylic acid is by reacting propene and steam with Pd
catalyst in a fuel cell. Pd acts as both an electrode and a catalyst in the fuel cell.
Propene is pumped in at the Pd electrode, while air (as a source of oxygen) is
pumped in at the Pt electrode. The cell is operated in the gas phase at 365 K and
atmospheric pressure. Electricity is generated during the reaction, making the process
economically viable.

161.9 J K1 mol1

External circuit

(i) Define the term standard enthalpy change of combustion of methanol.


acrylic acid

H2 O

(ii) Explain the significance of the sign of the standard entropy change of
combustion of methanol.
Cation
exchange
membrane
(H3PO4 in
silica wool)

(iii) Using the data given above, calculate the change in standard Gibbs free
energy for the combustion reaction.
[5]
[Total: 20]

propene,
H2O

O2

Pd
electrode

Pt
electrode

(i) Write half-equations for the reactions occurring at the cathode and the anode in
the fuel cell. State the polarities of each electrode.
(ii) The fuel cell is capable of producing an Ecell of +0.70 V under the conditions of
operation. Given that the electrode potential of the acrylic acid/propene half cell
is +0.53 V under these conditions, calculate the electrode potential of the
O2/H2O half cell under the same conditions.
(iii) With reference to your answer in (ii) and the Data Booklet, use Le Chateliers
Principle to determine if the operation temperature of the fuel cell should be
increased.
[6]
H2 Chemistry 9647/03 NYJC J2/2015 Prelim

[Turn over
H2 Chemistry 9647/03 NYJC J2/2015 Prelim

5
(c)

The electricity generated by the fuel cell during the reaction was used to electroplate
silica disks with Pt as part of the manufacture of new fuel cells. The electrolyte used
was a solution of [Pt(NH3)4]2+ (aq).

Soil is a complex mixture containing organic matter, oxides, carbonates and many silicate
materials.
(a)

(i) Draw a diagram to illustrate the electroplating set-up. You can use a battery
symbol ( ) to represent the fuel cell.

(i) Draw a dot-and-cross diagram to show the bonding in SiO44 and state its
shape.

(ii) The [Pt(NH3)4]2+ complex reacts to form platinum metal during the
electroplating process. Write a half-equation for this reaction.

(ii) Two SiO44 units are joined together through the formation of Si-O-Si covalent
bonds. Draw the resulting structure when two SiO44 units combine together. [3]

(iii) Suggest with reason, a value for E([Pt(NH3)4]2+/Pt).

(b)

(iv) A silica disk of surface area 2.00 cm2 was coated with a 0.40 cm thick layer of
Pt. Calculate the mass of acrylic acid generated from the fuel cell during this
plating process. (Density of Pt is 21.5 g cm-3)
[6]
(d)

Silicate minerals are generally based on the silicate, SiO44, structural unit.

A solution of carbon dioxide dissolved in water can be regarded as an equilibrium


mixture of carbonic acid, hydrogencarbonate ions and hydrogen ions. Given that the
concentration of carbonic acid in soil is 1.33 x 103 mol dm3 and the acid dissociation
constant, Ka for carbonic acid is 4.5 x 107 mol dm3.

Acrylic acid has a pKa of 4.25, while propanoic acid has a pKa of 4.72.

Write an expression of Ka for the weak acid formed from a solution of carbon dioxide
and hence determine the pH of a solution of carbon dioxide in the soil.
[3]

(i) Explain why the pKa of acrylic acid is lower than that of propanoic acid.
(c)

Aluminum ions generated by weathering of rock materials is another cause of acidity


in soil. Using equations, explain how the presence of aluminum ions enhance acidity
of the soil.
[2]

(d)

The decomposition of organic matter also leads to an increase in soil acidity.


Humic acids such as vanillic acid and ferulic acid are formed in the process.

Acrylic acid can be readily polymerised to form poly(acrylic acid), a useful material for
making thickening agents and super absorbent polymers for use in diapers.

CO2H
n H2C=CHCO2H

CH2

The most important natural soil acidification process is respiration.

C
H

OH
OCH 3

OH

Poly(acrylic acid) is a very effective thickening agent. In aqueous solutions of


pH > 4, the polymer chain stretches out from a random coil formation. This causes the
polymer to take up a lot more space, resisting the flow of the solvent molecules
around it, hence, making the solution viscous.
(ii) Explain why the chain stretches out at pH > 4.
(iii) Explain why at pH > 4, the solution of poly(acrylic acid) loses its viscous nature
when some sodium chloride is added to the solution.
[4]
[Total: 20]

OCH 3
CH
CH
COOH

COOH

vanillic acid

ferulic acid

(i) Suggest a simple chemical test to distinguish between the two acids.
(ii) With the aid of an equation, explain how vanillic acid and its conjugate base
can regulate the pH of the soil when it becomes too acidic.
(iii) The pH of a sample of soil is 5.5 and the ratio of the concentration of vanillic
acid to its conjugate base in soil is 0.102. Calculate the value for Ka of vanillic
acid.
[6]

H2 Chemistry 9647/03 NYJC J2/2015 Prelim

[Turn over
H2 Chemistry 9647/03 NYJC J2/2015 Prelim

7
(e)

Crushed limestone, CaCO3, and hydrated lime, Ca(OH)2, are used as liming materials
to neutralise the excessive acids in the soil and create optimum soil conditions for
crop growth.
Relevant Ksp values at 25 C are given in the table.

CaCO3
Ca(OH)2

This question is about iron as a transition metal and its compounds.


(a) Transition metals are often good homogenous and heterogeneous catalysts.
Finely divided Fe is used as a catalyst in the manufacture of ammonia in Haber
process.
H = 92 kJ mol1

Ksp

N2(g) + 3H2(g) 2NH3(g)

2.8 x 109
5.6 x 106

(i) State the temperature and pressure for the Haber process.
(ii) Why is iron needed to catalyse the reaction for Haber process?

(i) By considering the solubility of the salts, suggest why Ca(OH)2 is a better
liming material.
(ii) Suggest why the decomposition of organic matter is preferred over liming to
control pH.
[4]
(f)

Effective growth of plants depends on the ability of the soil to retain ions for
subsequent use. This ability is referred to as a soils cation exchange capacity.
It can be quantified as the number of milligrams of hydrogen ions that can be held at
cation exchange sites by 100 g of soil.
A 10.0 g sample of soil is first treated so that all its cation exchange sites become
saturated with hydrogen ions. These ions are then released and made into a 250 cm 3
aqueous solution. A 25.0 cm3 sample of this solution is titrated against 0.010 mol dm3
sodium hydroxide and 24.00 cm3 are required for neutralisation.
Calculate the cation exchange capacity of the soil.

(iii) Identify the type of catalysis involved and explain clearly why iron is a suitable
catalyst.
(iv)

Under certain conditions, when nitrogen and hydrogen are mixed in a 1:3 mole
ratio, there is 70% conversion of nitrogen to ammonia at equilibrium.
Write an expression for the equilibrium constant Kp, including units.

(v) Hence calculate the Kp value for this reaction if the total pressure of the
equilibrium mixture is 5 x 107 Pa.
[9]
(b) Anhydrous iron(III) chloride is made by passing chlorine gas over heated iron.
It can be used as a catalyst in the acylation of benzene, a process called
Friedel-Crafts acylation, to produce carbonyl compounds.

[2]
[Total: 20]

O
anhydrous FeCl3

C
Cl

Heat

O
C
R

Benzene undergoes Friedel-Crafts acylation with compound W to give compound X.


Upon warming with aqueous alkaline iodine, compound X gives yellow crystals but
not compound W. Compound X reacts with HCN and NaCN in the cold to give
compound Y, which gives compound Z on reaction with hydrogen in the presence of
platinum.
OH
C

CH 2NH 2

CH3

Suggest structures for W, X and Y and explain all the reactions involved.

H2 Chemistry 9647/03 NYJC J2/2015 Prelim

[Turn over
H2 Chemistry 9647/03 NYJC J2/2015 Prelim

[7]

10

(c) The stability constant, Kstab, of complex ions measures the stability of the complex.
The table below lists some iron complexes together with their colours and their
stability constants.
For the equilibrium,

K stab

2+

[Fe(SCN)(H2O)5 ]2+
[Fe(H2O)6 ]3+ [SCN]

Formula of complex
[Fe(H2O)6]2+(aq)
[Fe(SCN)(H2O)5]2+(aq)
[Fe(CN)6]4(aq)
[Fe(CN)6]3(aq)

Colour
pale green
blood red
pale yellow
orange yellow

(a) MnO2 is used as the catalyst for the decomposition of 20 cm3 of 0.10 mol dm3 H2O2
solution.
2H2O2(aq)

2H2O(l) + O2(g)

The rate of the reaction can be followed by measuring the volume of oxygen evolved at
different times. The following results were obtained where V t is the volume of oxygen
evolved after t minutes. [All gas volumes are measured at room temperature and
pressure].

[Fe(H2O)6] (aq) + SCN (aq) [Fe(SCN)(H2O)5] (aq) + H2O(l)


3+

Kstab
1 x 102
1 x 1024
1 x 1031

(i) By considering the strength of the ligands and the data given in the table,
explain why the aqua complexes of iron(II) appears to be pale green while
cyano complexes of iron(II) are yellow.
(ii) When sodium cyanide is added to a blood red solution containing
[Fe(SCN)(H2O)5]2+, a colour change is observed.

t
/ min
5.0
10.0
15.0
20.0
25.0
30.0
35.5
40.0

Vt
/ cm3
17.5
32.0
41.0
48.0
53.5
57.5
60.5
63.0

Vf Vt
/ cm3
46.5
32.0
23.0
16.0
10.5
6.5
3.5
1.0

rate
/ cm3 min-1
a
b
c
d
e
f
g
h

Using values of t and Vt, plot these data on suitable axes, showing all your working and
drawing clearly any construction lines on your graph. Use your graph to determine:
(i) the order of reaction with respect to [H2O2],

Using the data given in the table, predict and explain the final colour of the
solution and the predominant complex formed after the addition of sodium
cyanide. Suggest the type of reaction that has taken place.
[4]
[Total: 20]

(ii) the initial rate of decomposition of [H2O2], in mol dm3 min1.


(iii) Hence write the rate equation for the reaction. Calculate a value for the rate
constant. Include units in your answer.
(iv) When the H2O2 has completely decomposed, the final volume of oxygen
produced is denoted by Vf. Vf Vt and rate data were recorded.
State the significance of the term Vf Vt and hence explain why by plotting a
graph of rate against Vf Vt can verify the order of reaction with respect to
[H2O2] found in (a)(i).
[9]

H2 Chemistry 9647/03 NYJC J2/2015 Prelim

[Turn over
H2 Chemistry 9647/03 NYJC J2/2015 Prelim

11
(b) Manganese forms a carbonyl complex with the formula Mn 2(CO)10 in which each
manganese atom has coordination number 6.
(i) Draw the structure of the Mn2(CO)10 complex.
(ii) Treatment of the complex with sodium metal produces the compound
NaMn(CO)5. State the oxidation state of the manganese in this complex and
suggest its shape.
[3]

NANYANG JUNIOR COLLEGE


JC 2 PRELIMINARY EXAMINATION
Higher 2

CHEMISTRY

9647/03
21 September 2015

Paper 3 Free Response

(c) Explain the geometry around the nickel and state the oxidation state of nickel in the
following cases by drawing suitable complexes.
(i) Ni(NH3)2I2, exists in two forms, one of which has a dipole moment.

2 hours
Candidates answer Section A on the Question Paper
Candidates answer on separate paper.

(ii) K2Ni(NH3)2(CN)2 exists in only one form.

[5]

Additional Materials:

Answer Paper
Data Booklet
Graph Paper

(d) SiCl4 hydrolyses rapidly in water at room temperature.


READ THESE INSTRUCTIONS FIRST

(i) Write a balanced equation for the hydrolysis of SiCl4 in water.


(ii) The first step in the hydrolysis of SiCl4 proceeds via a SN2 mechanism to form a
trigonal bipyramidal intermediate.
Draw the structure of the intermediate formed when SiCl4 is hydrolysed.
(iii) The hydrolysis of CH3Cl also proceeds via SN2 mechanism. However, there was
no intermediate formed. Suggest a reason for this.
[3]
[Total: 20]

Write your name and class on all the work you hand in.
Write in dark blue or black pen on both sides of the paper.
You may use a soft pencil for any diagrams, graphs or rough working.
Do not use staples, paper clips, highlighters, glue or correction fluid.
Answer any four questions.
A Data Booklet is provided.
The use of an approved scientific calculator is expected, where appropriate.
You are reminded of the need for good English and clear presentation in your answers.
At the end of the examination, fasten all your work securely together.
The number of marks is given in brackets [ ] at the end of each question or part question.

For Examiners Use


Section A
1
2
3
4

This document consists of 11 printed pages

H2 Chemistry 9647/03 NYJC J2/2015 Prelim

[Turn over
[Turn over

2
Functional groups in a molecule must be protected in order to react in a desired way with
reagents used in a synthesis. For instance, alcohols undergo oxidation or nucleophilic
substitution reactions readily. To avoid these reactions, the alcohol can be protected by
converting it to an unreactive methoxymethyl ether (MOM), RO
methylchloromethyl ether (MOMCl), Cl

OCH3 , using

MOMCl

base
RO

RO
Step II

OCH3
MOM

(iv) Describe the mechanism of Step II for the protection process using MOMCl.
Type of reaction: Nucleophilic Substitution (SN2) [1]

Deprotection Process

RO

OCH3
MOM

(a)

H
C OCH3
ROH +
H
intermediate A

H2O

H
C

RO

O + CH3OH

Cl

RO

C
H

OCH3

Cl

RO

Cl

(i) Suggest why ROH is converted to RO in Step I.


The base added will deprotonate the alcohol to form the alkoxide ion which is a
stronger nucleophile. [1] The negatively charged alkoxide ion can better attack
the electron-deficient carbon in MOMCl, substituting the halogen atom to form
MOM.

[1 mark for reagents, transition state and products in correct geometry]


[1 mark for lone pair of electrons, charges, +/, arrows]
(v) State the oxidation state of carbon labelled in intermediate A.
0 [1]
+1
H

C
H
+1

H +1
x
xC
H

(ii) Brnsted-Lowry bases are usually used in Step I of the protection process.
Bulky organic bases such as N,N-diisopropylethylamine (DIPEA) is one
example.

H+(aq)

OCH3

OCH3

Step I

(iii) Explain why Step I does not occur if the base was changed to sodium
hydroxide.
Alkalis are not preferred as aliphatic alcohols are not acidic enough to be
deprotonated by alkalis. [1] The eletron donating alkyl group of ROH will
increase the electron density on the O atom of its conjugate base, making it
more negative, therefore destabilising it. Hence, ROH is weakly acidic.

OCH3 .

Protection Process
ROH

are more accessible as it contains less bulky alkyl groups hence they are
stronger nucleophiles.

x
x

H +1
+1

Compound F can be made from compound B in four steps, where the first step
involves the protection of alcohol using MOMCl.
COOH
Step III

DIPEA
Bulky organic bases such as DIPEA are preferred over
less bulky organic bases such as primary amine and
alkalis such as sodium hydroxide.

OH
B

MOMCl,
DIPEA

Step IV

Step V
KCN, CH3CH2OH
heat under reflux

Step VI
OH
F

(vi) Suggest reagents and conditions for Steps IV and VI, and the structures of
compounds C, D and E.

Explain why primary amine is unsuitable to be used as a base in Step I of the


protection process.
Less bulky organic bases are not preferred as they may act as nucleophiles
and attack MOMCl instead. [1] The lone pair of electrons on primary amines
H2 Chemistry 9647/03 NYJC J2/2015 Prelim

H2 Chemistry 9647/03 NYJC J2/2015 Prelim

5
(ii) Explain the significance of the sign of the standard entropy change of
combustion of methanol.

Step III

OH

MOMCl,
DIPEA

HCl(g), room temperature

Step IV

Step V

CN

OCH3

(iii) Using the data given above, calculate the change in standard Gibbs free
energy for the combustion reaction.
[5]

Cl

Ethanolic NaCN
O

heat under reflux

OCH3

OCH3

COOH

Hr = Hf (products) Hf (reactants)
= [2(393.5) + 4(285.8)] [2(238.7) + 3(0)] = 1452.8 kJ [1]
o

Gr = Hr TSr
= 1452.8 (298)(161.9/1000)(2) [1]
= 1356.3 1360 kJ [1]

HCl(aq) or H2SO4(aq)
Heat under reflux

Step VI

Since the amount of gaseous particles produced is less than the initial amount,
there is a decrease in the number of ways to arrange the gaseous particles,
giving rise to a decrease in disorder hence entropy. Hence, the standard
entropy change of decomposition reaction has a negative sign. [1]

H
+
OH

[Total: 20]
C

O + CH3OH

[1 mark for each R&C for Step IV & VI]


[1 mark for each intermediate C, D & E]
(vii) State the types of reaction for Steps IV, V and VI.

[15]

Step IV: Electrophilic addition [1]


Step V: Nucleophilic substitution [1]
Step VI: Acidic hydrolysis [1]
(b) Methanol is currently studied as a potential green fuel that greatly reduces the
pollution to the environment.
2CH3OH(l) + 3O2(g) 2CO2(g) + 4H2O(l)
o

238.7 kJ mol1

393.5 kJ mol1

285.8 kJ mol1

161.9 J K1 mol1

Hf CH3OH(l)
Hf CO2(g)
Hf H2O(l)
Sc CH3OH(l)

(i) Define the term standard enthalpy change of combustion of methanol.


It is the energy evolved when one mole of methanol is completely burnt in
excess oxygen under standard conditions. [1]

H2 Chemistry 9647/03 NYJC J2/2015 Prelim

H2 Chemistry 9647/03 NYJC J2/2015 Prelim

6
2

Acrylic acid is used in the manufacture of a large variety of plastics, paint formulations, and
polymer coatings. More than 1000 kilotonnes of acrylic acid are produced for industrial use
in a year.
O

1st method:
Cl

In step I, the alkene will undergo electrophilic addition with Cl2 to form

Cl

OR

OH
acrylic acid
(a)

The yield is low as free radical substitution can take place at other carbon in
step I (produces isomeric product/is random).

(i) Suggest a 3-step synthesis of acrylic acid starting


State all reagents and conditions and draw all intermediates.

from

propene.
Cl

and
Cl

step I

Cl

step II

OH

may be produced as well

2nd method:
The yield is low as oxidation of 2o alcohol can take place in step II to produce

step III

OH

Step I: limited Cl2, UV light


Step II: NaOH (aq), heat under reflux
Step III: K2Cr2O7 (aq), H2SO4 (aq), heat under reflux
(KMnO4 cannot be used as an alkene is present)

OH

or
OH

O
O

may be produced instead.

3rd method:
The yield is low as the major product of electrophilic addition in step I

OR

Br

(predicted by Markonikovs rule) is

OH

is produced instead.

step I

OH

step II

OH

step III

(b)
OH

OH

Another method of synthesising acrylic acid is by reacting propene and steam with Pd
catalyst in a fuel cell. Pd acts as both an electrode and a catalyst in the fuel cell.

OH

Step I: KMnO4, H2SO4(aq), cold


Step II: K2Cr2O7/KMnO4, H2SO4(aq), heat under reflux
Step III: conc H2SO4, 170 oC or Al2O3, 350 oC
Order of steps II and III can be reversed.

Propene is pumped in at the Pd electrode, while air (as a source of oxygen) is


pumped in at the Pt electrode. The cell is operated in the gas phase at 365 K and
atmospheric pressure. Electricity is generated during the reaction, making the process
economically viable.

OR
O
O

step I

OH

step II

OH

step III
OH

Br

Br

Step I: Br2 (aq), room temp


Step II: ethanolic NaOH, heat under reflux
Step III: K2Cr2O7/KMnO4, H2SO4(aq), heat under reflux
It is more ideal to do steps II and III in order above as carboxylic acid reacts
with NaOH.
[1] for each step and corresponding intermediate.
(ii) Explain one disadvantage of this method of synthesis.
H2 Chemistry 9647/03 NYJC J2/2015 Prelim

[4]
H2 Chemistry 9647/03 NYJC J2/2015 Prelim

9
(c)

External circuit

acrylic acid

The electricity generated by the fuel cell during the reaction was used to electroplate
silica disks with Pt as part of the manufacture of new fuel cells. The electrolyte used
was a solution of [Pt(NH3)4]2+ (aq).

H2 O

(i) Draw a diagram to illustrate the electroplating set-up. You can use a battery
symbol ( ) to represent the fuel cell.
Cation
exchange
membrane
(H3PO4 in
silica wool)
propene,
H2O

O2

Silica disk with Pt or graphite coating

Pd
electrode

(b)

Pt
electrode

(i) Write half-equations for the reactions occurring at the cathode and the anode in
the fuel cell. State the polarities of each electrode.
(Pt) Cathode (+ve): O2 + 4H+ + 4e 2H2O [1]
(Pd) Anode (-ve): CH2CHCH3 + 2H2O CH2CHCOOH + 6H+ + 6e [1]
[1] for correct polarity
(ii) The fuel cell is capable of producing an Ecell of +0.70 V under the conditions of
operation. Given that the electrode potential of the acrylic acid/propene half cell
is +0.53 V under these conditions, calculate the electrode potential of the
O2/H2O half cell under the same conditions.
Ecell = E[R] E[O]
E[R] = 0.70 + 0.53 = +1.23 V [1]
(iii) With reference to your answer in (ii) and the Data Booklet, use Le Chateliers
Principle to determine if the operation temperature of the fuel cell should be
increased.
[6]
O2 + 4H+ + 4e H2O
E(O2/H2O) = +1.23V at 298K and at 365K (remains unchanged)
E(O2/H2O) is independent of temperature. [1]

Complete set up with Pt electrode, [Pt(NH3)4]2+ electrolyte and silica disk at


negative terminal. [1]
(ii) The [Pt(NH3)4]2+ complex reacts to form platinum metal during the
electroplating process. Write a half-equation for this reaction.
[Pt(NH3)4]2+ + 2e Pt + 4NH3 [1]
(must be full arrow)
(ii) Suggest with reason, a value for E([Pt(NH3)4]2+/Pt).
2H2O + 4e H2 + 4OH 0.83 V
E([Pt(NH3)4]2+/Pt) has to be more +ve / less ve than 0.83 V [1]
otherwise H2O will be prefentially reduced. [1]
(iii) A silica disk of surface area 2.00 cm2 was coated with a 0.40 cm thick layer of
Pt. Calculate the mass of acrylic acid generated from the fuel cell during this
plating process. (Density of Pt is 21.5 g cm-3)
[6]
Mass of Pt coated = (21.5)(0.40 x 2.00) = 17.2 g
Amount of electrons passed during coating = (17.2 / 195)(2) = 0.1764 mol [1]

The operation temperature of the fuel cell need not be increased. [1]
(CH2CHCH3 + 2H2O CH2CHCOOH + 6H+ + 6e)
6 mol of electrons are required per mol of acrylic acid generated.
Amount of acrylic acid = 0.1764 / 6 = 0.02940 mol
Mass of acrylic acid = 0.02940 x 72.0 = 2.12 g [1]

H2 Chemistry 9647/03 NYJC J2/2015 Prelim

H2 Chemistry 9647/03 NYJC J2/2015 Prelim

10
(d)

11
3

Acrylic acid has a pKa of 4.25, while propanoic acid has a pKa of 4.72.
(i) Explain why the pKa of acrylic acid is lower than that of propanoic acid.
-

Soil is a complex mixture containing organic matter, oxides, carbonates and many silicate
materials.
(a) Silicate minerals are generally based on the silicate, SiO44, structural unit.
(i) Draw a dot-and-cross diagram to show the bonding in SiO44 and state its
shape.

In
, the p orbital of oxygen overlaps with both the electron cloud of
the C=O and C=C bonds. [1] This disperses the ve charge to a greater extent,
stabilising the conjugate base, making acrylic acid a stronger acid (with a lower
pKa). [1]
O

Acrylic acid can be readily polymerised to form poly(acrylic acid), a useful material for
making thickening agents and super absorbent polymers for use in diapers.

Tetrahedral shape
1m for correct shape and 1m for correct dot-and-cross

CO2H
n H2C=CHCO2H

CH2

(ii) Two SiO44 units are joined together through the formation of Si-O-Si covalent
bonds. Draw the resulting structure when two SiO44 units combine together. [3]

C
H

Poly(acrylic) acid is a very effective thickening agent. In aqueous solutions of


pH > 4, the polymer chain stretches out from a random coil formation. This causes the
polymer to take up a lot more space, resisting the flow of the solvent molecules
around it, hence, making the solution viscous.
(ii) Explain why the chain stretches out at pH > 4.
At pH > 4, most of the COOH groups on poly(acrylic acid) are deprotonated to
form COO. The negative charges on the chain are in close proximity and they
repel each other, causing the chain to spread out. [1]
(iii) At pH > 4, the solution of poly(acrylic acid) loses its viscous nature when some
sodium chloride is added to the solution. Explain why.
[4]
The Na+ ions form ionic bonds with the COO groups via ionic interactions.
There is no longer any repulsion between the COO groups and the chain
collapses, causing the solution to lose its viscous nature. [1]
[Total: 20]

H2 Chemistry 9647/03 NYJC J2/2015 Prelim

(b)

The most important natural soil acidification process is respiration.


A solution of carbon dioxide dissolved in water can be regarded as an equilibrium
mixture of carbonic acid, hydrogencarbonate ions and hydrogen ions. Given that the
concentration of carbonic acid in soil is 1.33 x 103 mol dm3 and the acid dissociation
constant, Ka for carbonic acid is 4.5 x 107 mol dm3.
Write an expression of Ka for the weak acid formed from a solution of carbon dioxide
and hence determine the pH of a solution of carbon dioxide in the soil.
[3]

C
C

[H+]2 = 4.5 x 10-7 x 1.33 x 10-3 = 5.985 x 10-10


[H+] = 2.446 x 10-5 [1]

pH = 4.61 [1]

H2 Chemistry 9647/03 NYJC J2/2015 Prelim

12
(c)

13
(ii) With the aid of an equation, explain how vanillic acid and its conjugate base
can regulate the pH of the soil when it becomes too acidic.

Aluminum ions generated by weathering of rock materials are another cause of acidity
in soil. Using equations, explain how the presence of aluminum ions enhance acidity
of the soil.
[2]

OH

OH
OCH3

The aluminum ions Al3+ undergoes hydration to give the hydrated aluminum ions.
Al3+ (aq) + 6H2O (l) [Al(H2O)6]3+ (aq)
3+

+
COO

2+

[Al(H2O)6] then undergoes further hydrolysis to give [Al(H2O)5OH] (aq)


[Al(H2O)6]3+ (aq) + H2O (l)
[Al(H2O)5OH]2+ (aq) + H3O+ (aq)
The highly polarizing Al3+due to its high charge density is able to polarise the O-H
bond in H2O and hence weakening the O-H bond, causing H+ to be released to give
an acidic solution.

COOH

When the soil becomes too acidic, the excess H+ ions are removed by the large
reservoir of the conjugate base of vanillic acid present, regulating the pH of the
soil.
[1] for explanation
[1] for equation

[1] for explanation


[1] for equation
(d)

OCH3

(iii) The pH of a sample of soil is 5.5 and the ratio of the concentration of vanillic
acid to its conjugate base in soil is 0.102. Calculate the value for K a of vanillic
acid.
[6]

The decomposition of organic matter also leads to an increase in soil acidity. Humic
acids such as vanillic acid and ferulic acid are formed in the process.

con ugate base of vanillic acid


a

OH

OCH 3

pKa = 4.508 [1]


Ka = 3.10 x 10-5 mol dm-3 [1]

OH
OCH 3

CH

Crushed limestone, CaCO3, and hydrated lime, Ca(OH)2, are used as liming materials
to neutralise the excessive acids in the soil and create optimum soil conditions for
crop growth.

COOH

COOH

Relevant Ksp values at 25 C are given in the table.

vanillic acid

ferulic acid

CH

(i) Suggest a simple chemical test to distinguish between the two acids.
KMnO4(aq), H2SO4(aq), heat [1]
Ferulic acid turns purple KMnO4 colourless. Effervescence of CO2 gas
observed and gives a white ppt with limewater. [1]
For vanillic acid, the purple KMnO4 remains and no effervescence is observed.

(e)

CaCO3
Ca(OH)2

(i) By considering the solubility of the salts, suggest why Ca(OH)2 is a better
liming material.
Let solubility of Ca(OH)2 in mol dm-3 be x
Ca(OH)2
Ca2+ + 2OH
Ksp (Ca(OH)2)
5.6 x 10-6
4x3
x

H2 Chemistry 9647/03 NYJC J2/2015 Prelim

Ksp
2.8 x 10 mol2 dm-6
5.6 x 10-6 mol3 dm-9
-9

= [Ca2+] [OH]2
=
(x) (2x)2
= 5.6 x 10-6
= 1.12 x 10-2 mol dm-3 [1]

H2 Chemistry 9647/03 NYJC J2/2015 Prelim

14

15
4

Let solubility of CaCO3 in mol dm-3 be y


CaCO3
Ca2+ + CO3

(a) Transition metals are often good homogenous and heterogeneous catalysts. Finely
divided Fe is used as a catalyst in the manufacture of ammonia in Haber process.

N2(g) + 3H2(g) 2NH3(g)

92 kJ mol1

Ksp (CaCO3) = [Ca2+] [CO3 ]


2.8 x 10-9
=
(y) (y)
2
y
= 2.8 x 10-9
y
= 5.29 x 10-5 mol dm-3 [1]

(i) State the temperature and pressure for the Haber process.

Ca(OH)2 is a better liming material due to its higher solubility. [1]

(ii) Why is iron needed to catalyse the reaction for Haber process?

500 oC, 250 atm [1]

(ii) Suggest why the decomposition of organic matter is preferred over liming to
control pH.
[4]

For the effective growth of plants, apart from controlling the pH of the soil, it requires a
sufficient supply of nutrients and also on the ability of the soil to retain ions for
subsequent use This ability is referred to as a soils cation exchange capacity. It can
be quantified as the number of milligrams of hydrogen ions that can be held at cation
exchange sites by 100 g of soil.
A 10.0 g sample of soil is first treated so that all its cation exchange sites become
saturated with hydrogen ions. These ions are then released and made into a 250 cm3
aqueous solution. A 25.0 cm3 sample of this solution is titrated against 0.010 mol dm3
sodium hydroxide and 24.00 cm3 are required for neutralisation.
Calculate the cation exchange capacity of the soil.

The decomposition of organic matter provides buffering action in the long run
but liming simply neutralizes the acid present. [1]
(f)

This question is about iron as a transition metal and its compounds.

[2]

(iii) Identify the type of catalysis involved and explain clearly why iron is a suitable
catalyst.

Amount of hydrogen ions in 25.0 cm3 sample


= 0.010 x 24.00/1000 = 2.40 x 10-4 mol
Amount of hydrogen ions in 250 cm3 solution
= 2.40 x 10-4 x 250/25.0 = 2.40 x 10-3 mol [1]
Mass of hydrogen ions in 10.0 g of soil
= 2.40 x 10-3 x 1.0 = 2.40 x 10-3 g = 2.40 mg
Cation exchange capacity of the soil = 2.40 x 100/10.0 = 24 mg per 100 g of soil. [1]

[Total: 20]

By Le Chateliers Principle, low temperature will cause the system to react


in a way to increase the temperature.
The exothermic forward reaction is favoured and position of equilibrium
shifts to the right to produce more products.
However, at low temperatures, rate of reaction is slow, and the system will
take a long time to reach equilibrium.
Hence an optimal temperature of 500 oC and a catalyst is used to ensure
that both yield and rate are high.
4 points [2]
3 points [1]

Transition elements are good heterogeneous catalysts because they


possess partially filled d subshells. [1]
The d electrons are available for the formation of weak temporary bonds
with the reactant molecules also,
The energetically low-lying vacant orbitals can be used to accommodate
electrons pairs from reactant molecules, resulting in bond formation.
2 points [1]

(iv) Under certain conditions, when nitrogen and hydrogen are mixed in a 1:3 mole
ratio, there is 70% conversion of nitrogen to ammonia at equilibrium.
Write an expression for the equilibrium constant Kp, including units.
Kp =

PNH3 2
PN2 PH2 3

Units: Pa2

[1]

(v) Hence calculate the Kp value for this reaction if the total pressure of the
equilibrium mixture is 5 x 107 Pa.
[9]
Let the initial no. of mol of N2 be x.

H2 Chemistry 9647/03 NYJC J2/2015 Prelim

H2 Chemistry 9647/03 NYJC J2/2015 Prelim

16
N2(g)
Initial no.
of mol
Change in
no. of mol
Eqm no of
mol

17
+

3H2(g)

2NH3(g)

3x

0.7x

2.1x

+1.4x

0.3x

0.9x

(b) Anhydrous iron(III) chloride is made by passing chlorine gas over heated iron.
It can be used as a catalyst in the acylation of benzene, a process called FriedelCrafts acylation, to produce carbonyl compounds.
O

1.4x

Total amount at equilibrium = 0.3x + 0.9x + 1.4x = 2.6x mol


[1] for ICE table
At equilibrium
0.3x
(5 107 ) 5.769 107 Pa
2.6x
6
0.9 x
PH2 =
(5 107 ) = 1.730 x 10 Pa
2.6x
7
1.4 x
PNH3 =
(5 107 ) = 2.623 x 10 Pa [1] for partial pressures
2.6x

anhydrous FeCl3

C
Cl

Heat

Benzene undergoes Friedel-Crafts acylation with compound W to give compound X.


Upon warming with aqueous alkaline iodine, compound X gives yellow crystals but
not compound W. Compound X reacts with HCN and NaCN in the cold to give
compound Y, which gives compound Z on reaction with hydrogen in the presence of
platinum.

PN2 =

Kp =

[1]

PNH3 2
PN2 PH2

(2.623 106 )2
(5.769 106 )(1.730 106 )3

OH
C

CH 2NH 2

CH3

= 2.42 x 1014 Pa2

Suggest structures for W, X and Y and explain all the reactions involved.

[7]

correct Kp value
O
C
H3C

Cl

W
[1]

H2 Chemistry 9647/03 NYJC J2/2015 Prelim

OH

O
C

CH3

X
[1]

Y
[1]

CN

CH3

X contains a methyl ketone group as it undergoes mild oxidation with I2(aq) and
NaOH(aq) to give a yellow ppt of CHI3. [1]

X contains a carbonyl function group since it undergoes nucleophilic addition with


HCN and NaCN, [1]

W does not contain the methyl ketone or methyl alcohol group as it does not give
yellow ppt. [1]

Compound Y undergoes reduction to give Z with H2 and Pt. Thus Y contains the
cyanohydrin group. [1]

H2 Chemistry 9647/03 NYJC J2/2015 Prelim

18

19
[Total: 20]

(c) The stability constant, Kstab, of complex ions measures the stability of the complex.
The table lists some iron complexes together with their colours and their stability
constants.
For the equilibrium,
[Fe(H2O)6]3+(aq) + SCN(aq) [Fe(SCN)(H2O)5]2+(aq) + H2O(l)
K stab

[Fe(SCN)(H2O)5 ]2+
[Fe(H2O)6 ]3+ [SCN ]

Formula of complex
[Fe(H2O)6]2+(aq)
[Fe(SCN)(H2O)5]2+(aq)
[Fe(CN)6]4-(aq)
[Fe(CN)6]3-(aq)

Colour
pale green
blood red
pale yellow
orange yellow

Kstab
1 x 102
1 x 1024
1 x 1031

(i) By considering the strength of the ligands and the data given in the table,
explain why the aqua complexes of iron(II) appears to be pale green while
cyano complexes of iron(II) are yellow.

A yellow colour in cyano complexes is due to absorption of violet light


around 400nm, and a green colour in aqua complexes is due to absorption
of red light around 600nm. [1]

CN is a strong ligand, thus its complexes are expected to have larger E


than aqua complexes. The larger E corresponds to a shorter wavelength
absorbed by the d electrons, thus the cyano complexes appear as yellow.
[1]

(ii) When sodium cyanide is added to a blood red solution containing


[Fe(SCN)(H2O)5]2+, a colour change is observed.
Using the data given in the table, predict and explain the final colour of the
solution and the predominant complex formed after the addition of sodium
cyanide. What is the type of reaction that has taken place?
[4]

Blood red solution turns orange yellow. [Fe(CN)6]3- forms after the addition
of NaCN. Since Kstab of [Fe(CN)6]3- is larger than [Fe(SCN)(H2O)5]2+,
[Fe(CN)6]3- is the more stable complex. [1] final colour + reference to Kstab

Ligand exchange will occur. [1] type of reaction

H2 Chemistry 9647/03 NYJC J2/2015 Prelim

H2 Chemistry 9647/03 NYJC J2/2015 Prelim

20
5

21

(a) MnO2 is used as the catalyst for the decomposition of 20 cm 3 of a 0.267 mol dm3
H2O2 solution.
2H2O2(aq)

(i) the order of reaction with respect to [H2O2],

2H2O(l) + O2(g)

The rate of the reaction can be followed by measuring the volume of oxygen evolved
at different times. The following results were obtained where Vt is the volume of
oxygen evolved after t minutes. [All gas volumes are measured at room temperature
and pressure].
t
/ min
5.0
10.0
15.0
20.0
25.0
30.0
35.5
40.0

Vf Vt
/ cm3
46.5
32.0
23.0
16.0
10.5
6.5
3.5
1.0

Vt
/ cm3
17.5
32.0
41.0
48.0
53.5
57.5
60.5
63.0

rate
/ cm3 min-1
a
b
c
d
e
f
g
h

Since half-life is constant at 10 min, reaction is first order with respect to [H2O2].
[1]
(ii) the initial rate of decomposition of [H2O2], in mol dm3 min1.
Initial rate of volume of oxygen formed = 44/10 = 4.40 cm 3 min1
[1] for showing gradient at t = 0 on graph and using that to calculate initial rate
(coordinate of 44,10 was chosen)
Initial rate of amount of oxygen formed =

= 1.83 x 104 mol min1

Initial rate of amount of hydrogen peroxide reacted


= 2 x 1.83 x 104 = 3.67 x 104 mol min1

Initial rate of decrease of [H2O2] =


Using values of t and Vt, plot these data on suitable axes, showing all your working
and drawing clearly any construction lines on your graph. Use your graph to
determine:

= 0.0183 mol dm3 min1 [1]

(iii) Hence write the rate equation for the reaction. Calculate a value for the rate
constant. Include units in your answer.

Vt / cm3
Rate = k[H2O2] [1]

Vf = 64
70

Initial concentration of [H2O2] = 0.267 mol dm3


60

Rate constant k =

= 0.0689 min1 (units must be correct) [1]

50

Alternative method:
t1/2 = ln2 / k
10 = ln2 / k
k = 0.0693 min1 (note that k should be similar regardless of method used)

40

30

(iv) When the H2O2 has completely decomposed, the final volume of oxygen
produced is denoted by Vf. Vf Vt and rate data were recorded.

20

State the significance of the term Vf Vt and hence explain why by plotting a
graph of rate against Vf Vt can verify the order of reaction with respect to
[H2O2] found in (a)(i).
[9]

10
t1/2 = 10 min

t1/2 = 10 min

t / min

0
0

10

15

20

25

30

35

40

45

[1] for plotting graph of Vt / cm3 against t / min (with labelled axis and units)
[1] for showing 2 correct half-lives on graph
H2 Chemistry 9647/03 NYJC J2/2015 Prelim

Vf Vt is proportional to [H2O2] remaining. [1]


When a graph of rate against Vf Vt is plotted, reaction is first order with
respect to hydrogen peroxide if an upward-sloping straight line graph that

H2 Chemistry 9647/03 NYJC J2/2015 Prelim

22

23

passes through the origin is obtained. [1]


(b) Manganese forms a carbonyl complex with the formula Mn 2(CO)10 in which each
manganese atom has coordination number 6.

PR3

Ni

(i) Draw the structure of the Mn2(CO)10 complex.

CO
Mn

OC

NC
PR3

NC

CO
OC

CO
CO
Mn

1m for correct shape and 1m for correct oxidation state.


CO

(d) SiCl4 hydrolyses rapidly in water at room temperature.

OC
CO

CO

[1]

(i) Write a balanced equation for the hydrolysis of SiCl4 in water.

(ii) Treatment of the complex with sodium metal produces the compound
NaMn(CO)5. Calculate the oxidation state of the manganese in this complex
and suggest its shape.
[3]
Each CO is neutral in charge. And since anion is [Mn(CO)5], therefore
x+ 5(0) = 1, x = 1 [1]
The complex will acquire a square pyramidal shape [since the MnMn bond
was broken and each Mn atom acquired an electron to form a lone pair plus 5
bond pairs of MnCO]. [1]

SiCl4 (l) + 2H2O (l) SiO2 (s) + 4HCl (g)


or
SiCl4 (l) + 4H2O (l) Si(OH)4 (s) + 4HCl (g)
(ii) The first step in the hydrolysis of SiCl4 proceeds via a SN2 mechanism to form
a trigonal bipyramidal intermediate.
Draw the structure of the intermediate formed when SiCl4 is hydrolysed.
H2O

(c) Explain the geometry around the nickel and state the oxidation state of nickel in the
following cases by drawing suitable complexes.
(i) Ni(NH3)2I2, exists in two forms, one of which has a dipole moment.

Cl
Si
Cl

Cl
[1]

PR3

PR3

Ni
I

Cl

(iii) The hydrolysis of CH3Cl also proceeds via SN2 mechanism. However, there
was no intermediate formed. Suggest a reason for this.
[3]

Ni
PR3

With dipole moment

PR3

Si has available d orbitals that can incorporate the H 2O ligand forming a


5-coordinate intermediate (or whereas C can only form the transition state in a
Sn2 mechanism as it does not have available d-orbitals to incorporate the H2O
ligand). [1]

Without dipole moment

Oxidation state of Ni is +2 [1]

[Total: 20]

1 mark each for each correct diagram and correctly stating the dipole moments
[possibly can ignore stating of dipole moments as drawing of correct isomers
shows understanding]

(ii) K2Ni(NH3)2(CN)2 exists in only one form.

[5]
2-

H2 Chemistry 9647/03 NYJC J2/2015 Prelim

H2 Chemistry 9647/03 NYJC J2/2015 Prelim

Vous aimerez peut-être aussi